Tai Lieu Boi Duong Doi Tuyen VN Du IMO 2010

You might also like

Download as pdf or txt
Download as pdf or txt
You are on page 1of 81

Ti liu bi dng i tuyn Vit Nam

tham d IMO 2010

Thng 6 - 2010

Vietnamese IMO Team Training Camp 2010

Mc lc
1.
2.
3.
4.
5.
6.

Cc phng php v k thut chng minh


Nguyn l chung v th
Gii phng trnh hm bng cch lp phng trnh
Cc bi ton ti u v h cc tp hp
V k thi chn i tuyn Vit Nam d thi IMO 2010
Bt ng thc: Mt s v d v bi tp chn lc

2 | Trn Nam Dng 6/2010

2
42
50
63
69
80

Vietnamese IMO Team Training Camp 2010

Cc phng php v k thut chng minh


Trong ton hc cng nh trong cuc sng, cn bit:
Linh hot x l tnh hung, chn la phng n ti u
Trn Nam Dng
Trng i hc KHTN Tp HCM
Cc nh l ton hc pht biu v cc tnh cht ca cc i tng ton hc v mi quan
h gia chng. V nhng khng nh ny cn c chng minh xut pht t cc tin ,
cc nh l v tnh cht c chng minh trc . V thc hin bc chng minh,
ta cn c nhng quy tc suy din chng minh l cht ch v mt ton hc.
Vi cc bi ton Olympic cng vy, yu cu chng minh mt kt qu no lun hin
din, ngay c trong nhng bi khng c cm t chng minh rng. Chng hn gii
phng trnh x3 3x + 1 = 0 c th ta s phi chng minh tt c cc nghim ca chng
thuc on [-2, 2], gii phng trnh hm f(x2 + f(y)) = f2(x) + y c th ta s phi
chng minh f l ton nh ...
Bi vit ny ni v hai phng php v mt s k thut chng minh c bn: chng minh
phn chng, chng minh quy np, chng minh phn chng, dng mnh phn o,
phn v d nh nht, v d v phn v d, s dng nguyn l Dirichlet, nguyn l cc hn,
nguyn l bt bin, s dng t mu, m bng hai cch, sp xp th t
Cch tip cn ca chng ta l s thng qua cc v d ni v cc phng php v k
thut. y s ch c cc nhn xt, bnh lun, cc nguyn tc chung ch khng c
trnh by h thng nh mt l thuyt.
Bi vit ny u tin c vit v trnh by trong chng trnh Gp g g Ton hc
2010, c t chc vo thng 1 nm 2010, sau c b sung, honh chnh v trnh
by ti Hi ngh khoa hc Cc chuyn chuyn Ton v ng dng t chc ti Ba V,
thng 5/2010. Cui cng, chun b cho i tuyn Vit Nam thi Olympic Ton quc t,
bi vit c b sung thm cc phn v m bng hai cch, Nguyn l cc hn, Sp xp
th t v ng dng ca cc phng php v k thut chng minh trong bi ton Ti u t
hp.
1. Php chng minh phn chng
Mt s v d m u
Chng minh phn chng c th ni l mt trong nhng v kh quan trng ca ton hc.
N cho php chng ta chng minh s c th v khng c th ca mt tnh cht no , n
3 | Trn Nam Dng 6/2010

Vietnamese IMO Team Training Camp 2010


cho php chng ta bin thun thnh o, bin o thnh thun, n cho php chng ta l
lun trn nhng i tng m khng r l c tn ti hay khng. V d kinh in nht v
php chng minh phn chng thuc v Euclid vi php chng minh
nh l. Tn ti v s s nguyn t.
y, Euclid gi s ngc li rng tn ti hu hn s nguyn t p1, p2, , pn. ng xt
tch N = p1p2pn + 1. N phi c t nht 1 c s nguyn t p. Khi , do p1, p2, , pn l
tt c cc s nguyn t nn tn ti i sao cho p = pi. Nhng khi p | 1, mu thun.
Bi tp
1. Chng minh rng tn ti v s s nguyn t dng 4k+3.
2. Chng minh rng tn ti v s s nguyn t dng 4k+1.

Phng php xung thang


Mt chng minh ni ting khc bng phng php phn chng chnh l chng minh ca
Euler cho nh l nh Fermat vi trng hp n = 4.
nh l. Phng trnh x4 + y4 = z4 (1) khng c nghim nguyn dng.
ng gi s rng phng trnh (1) c nghim nguyn dng. Khi , theo nguyn l
cc hn, tn ti nghim (x0, y0, z0) vi x0 + y0 + z0 nh nht. Sau , bng cch s dng
cu trc nghim ca phng trnh Pythagore x2 + y2 = z2, ng i n s tn ti ca mt
nghim (x1, y1, z1) c x1 + y1 + z1 < x0 + y0 + z0. Mu thun.
Phng php ny thng c gi l phng php xung thang.
Bi tp
3. Chng minh rng phng trnh x3 + 3y3 = 9z3 khng c nghim nguyn dng.
4. Chng minh rng phng trnh x2 + y2 + z2 = 2xyz khng c nghim nguyn dng.

S dng mnh phn o


Chng minh s dng mnh phn o cng l mt phng n chng minh phn chng
hay c s dng. C s ca phng php l chng minh A  B, ta c th chng
minh B A . V mt bn cht th hai php suy din ny c v ging nhau, nhng trong
thc t th li kh khc nhau. Ta th xem xt 1 vi v d.
V d 1. Chng minh rng hm s f ( x) =

x
x2 +1

l mt n nh t R vo R.

4 | Trn Nam Dng 6/2010

Vietnamese IMO Team Training Camp 2010


V d 2. Chng minh rng nu (p-1)! + 1 l s nguyn t th p l s nguyn t.
Trong v d 1, r rng vic chng minh x1 x2 suy ra f(x1) f(x2) kh khn hn vic
chng minh f(x1) = f(x2) suy ra x1 = x2, d rng v mt logic, hai iu ny l tng
ng.
Trong v d 2, gn nh khng c cch no khc ngoi cch chng minh nu p l hp s, p
= r.s th (p-1)! + 1 khng chia ht cho p.
Bi tp.
5. Cho hm s f: R  R tho mn cc iu kin sau
1) f n iu ;
2) f(x+y) = f(x) + f(y) vi mi x, y thuc R.
6. Cho a, b, c l cc s thc khng m tho mn iu kin a2 + b2 + c2 + abc = 4. Chng minh rng a + b
+ c 3.

Phng php phn v d nh nht


Trong vic chng minh mt s tnh cht bng phng php phn chng, ta c th c
thm mt s thng tin b sung quan trng nu s dng phn v d nh nht. tng l
chng minh mt tnh cht A cho mt cu hnh P, ta xt mt c trng f(P) ca P l
mt hm c gi tr nguyn dng. By gi gi s tn ti mt cu hnh P khng c tnh
cht A, khi s tn ti mt cu hnh P0 khng c tnh cht A vi f(P0) nh nht. Ta s
tm cch suy ra iu mu thun. Lc ny, ngoi vic chng ta c cu hnh P0 khng c
tnh cht A, ta cn c mi cu hnh P vi f(P) < f(P0) u c tnh cht A.
V d 3. Cho ng gic li ABCDE trn mt phng to c to cc nh u nguyn.
a) Chng minh rng tn ti t nht 1 im nm trong hoc nm trn cnh ca ng gic
(khc vi A, B, C, D, E) c to nguyn.
b) Chng minh rng tn ti t nht 1 im nm trong ng gic c to nguyn.
c) Cc ng cho ca ng gic li ct nhau to ra mt ng gic li nh A1B1C1D1E1
bn trong. Chng minh rng tn ti t nht 1 im nm trong hoc trn bin ng gic li
A1B1C1D1E1.
Cu a) c th gii quyt d dng nh nguyn l Dirichlet: V c 5 im nn tn ti t nht
2 im X, Y m cp to (x, y) ca chng c cng tnh chn l (ta ch c 4 trng hp
(chn, chn), (chn, l), (l, chn) v (l, l)). Trung im Z ca XY chnh l im cn
tm.
Sang cu b) l lun trn y cha , v nu XY khng phi l ng cho m l cnh th
Z c th s nm trn bin. Ta x l tnh hung ny nh sau. rng nu XY l mt
cnh, chng hn l cnh AB th ZBCDE cng l mt ng gic li c cc nh c to
u nguyn v ta c th lp li l lun nu trn i vi ng gic ZBCDE, Ta c th
5 | Trn Nam Dng 6/2010

Vietnamese IMO Team Training Camp 2010


dng n bin chng minh qu trnh ny khng th ko di mi, v n mt lc no
s c 1 ng gic c im nguyn nm trong.
Tuy nhin, ta c th trnh by li l lun ny mt cch gn gng nh sau: Gi s tn ti
mt ng gic nguyn m bn trong khng cha mt im nguyn no (phn v d). Trong
tt c cc ng gic nh vy, chn ng gic ABCDE c din tch nh nht (phn v d nh
nht). Nu c nhiu ng gic nh vy th ta chn mt trong s chng. Theo l lun
trnh by cu a), tn ti hai nh X, Y c cp to cng tnh chn l. Trung im Z
ca XY s c to nguyn. V bn trong ng gic ABCDE khng c im nguyn no
nn XY phi l mt cnh no . Khng mt tnh tng qut, gi s l AB. Khi ng
gic ZBCDE c to cc nh u nguyn v c din tch nh hn din tch ng gic
ABCDE. Do tnh nh nht ca ABCDE (phn v d nh nht pht huy tc dng!) nn bn
trong ng gic ZBCDE c 1 im nguyn T. iu ny mu thun v T cng nm trong
ng gic ABCDE.
Bi tp
7. Gii phn c) ca v d 3.
8. (nh l Bezout) Chng minh rng nu (a, b) = 1 th tn ti u, v sao cho au + bv = 1.
9. Trn mt phng nh du mt s im. Bit rng 4 im bt k trong chng l nh ca mt t gic li.
Chng minh rng tt c cc im c nh du l nh ca mt a gic li.

Phn chng trong cc bi ton chng minh s khng tn ti


Phng php phn chng thng hay c s dng trong cc bi ton bt bin hoc bi
ton ph hnh chng minh s khng thc hin c. Sau y chng ta xem xt 2 v d
nh vy.
V d 4. Xt hnh vung 7 7 . Chng minh rng ta c th xo i mt phn cn li
khng th ph kn bng 15 qun trimino kch thc 1 3 v 1 qun trimino hnh ch L.
Ta chng minh rng nu b i mt gc trn bn tri th phn cn li khng th ph
c bng cc qun trimin cho.
lm iu ny, ta nh s cc vung nh sau
1
1
1
1
1
1
1

2
2
2
2
2
2
2

3
3
3
3
3
3
3

1
1
1
1
1
1
1

2
2
2
2
2
2
2

3
3
3
3
3
3
3

1
1
1
1
1
1
1
6 | Trn Nam Dng 6/2010

Vietnamese IMO Team Training Camp 2010


Khi , nhn xt rng 1 qun trimin kch thc 1 3 s che 3 s 1, 2, 3 (nu n nm
ngang) hoc 3 s ging nhau (nu n nm dc). Nh vy tng cc s m mt qun
trimin 1 3 che lun chia ht cho 3. Trong khi d thy qun trimin hnh ch L che
3 s c tng khng chia ht cho 3.
By gi gi s ngc li rng hnh vung 7 7 b i gc trn bn tri c th ph
c bng 15 qun trimin 1 3 v 1 qun trimin hnh ch L th theo l lun trn, tng
s cc s m cc qun trimin ny che s khng chia ht cho 3. iu ny mu thun v
tng cc s trn cc cn li bng
20 1 + 14 2 + 14 3 = 90
chia ht cho 3!
Mu thun trn chng t iu gi s l sai v ta c iu phi chng minh.
V d 5. Hnh trn c bi 5 ng knh thnh thnh 10 bng nhau. Ban u trong
mi c 1 vin bi. Mi ln thc hin, cho php chn 2 vin bi bt k v di chuyn chng
sang bn cnh, 1 vin theo chiu kim ng h v 1 vin ngc chiu kim ng h. Hi
sau mt s hu hn ln thc hin, ta c th chuyn tt c cc vin bi v cng 1 c
khng?
Nu lm th th chng ta s thy rng khng th thc hin c yu cu. Chng ta c th
cng lm l dn 9 vin bi v 1 , cn 1 vin bi khc th khng th dn c. Nhng lm
th no chng minh iu ny? Li gii ha ra l kh n gin. Ta s dng phn chng
kt hp vi bt bin.
Ta t mu cc bng hai mu en trng xen k nhau. Gi S l tng s vin bi nm cc
en th trng thi ban u ta c S = 5. Nu gi s ngc li rng ta c th a cc
vin bi v cng 1 th trng thi cui cng ny, ta s c S = 0 (nu ta dn cc vin bi
v mt trng) hoc S = 10 (nu ta dn cc vin bi v mt en).
By gi ta s thu c iu mu thun nu ta chng minh c qua cc ln thc hin th
tnh chn l ca S s khng thay i, tc l nu ban u S l s l th qua cc ln thc
hin, S s lun l s l (v s khng th bng 0 hoc bng 10).
Nu nhn xt rng cc en trng xen k nhau th iu m chng ta cn chng minh kh
hin nhin v chng ti xin dnh php chng minh chi tit cho bn c.
Bi tp
10. Hnh vung 5 x 5 b i gc trn bn tri. Chng minh rng c th ph phn cn li bng 8 qun
trimino hnh ch L nhng khng th ph c bng 8 qun trimino hnh ch kch thc 1 x 3. Tm tt c
cc gi tr k sao cho c th ph phn cn li bng k qun trimino 1 x 3 v 8-k trimino hnh ch L.
11. Xt hnh vung 7 7 . Tm tt c cc m nu ta xa i th phn cn li c th ph kn bng 15
qun trimino kch thc 1 3 v 1 qun trimino hnh ch L.

7 | Trn Nam Dng 6/2010

Vietnamese IMO Team Training Camp 2010


12. Trn vng trn ban u theo mt th t tu c 4 s 1 v 5 s 0. khong gia hai ch s ging
nhau ta vit s 1 v khong gia hai ch s khc nhau ta vit s 0. Cc s ban u b xo i. Hi sau
mt s ln thc hin nh vy ta c th thu c mt b gm 9 s 0?
13. Cho trc cc hm s f1(x) = x2 + 2x, f2(x) = x + 1/x, f3(x) = x2 - 2x . Cho php thc hin cc php
ton cng hai hm s, nhn hai hm s, nhn mt hm s vi mt hng s tu . Cc php ton ny c th
tip tc c thc hin nhiu ln trn fi v trn cc kt qu thu c. Chng minh rng c th thu c
hm s 1/x t cc hm s f1, f2, f3 bng cc s dng cc php ton trn nhng iu ny khng th thc
hin c nu thiu mt trong 3 hm f1, f2, f3.

Phn chng trong cc bi ton bt ng thc


Trong chng minh bt ng thc, phng php phn chng thng dng o iu
kin v kt lun vi nhau trong trng hp iu kin th phc tp, cn bt ng thc cn
chng minh th n gin.
V d 1. Chng minh rng nu a, b, c l cc s thc khng m tha mn iu kin a2 + b2
+ c2 + abc = 4 th a + b + c 3.
V d 2. (IMO 2001) Cho a, b, c l cc s thc dng. Chng minh rng ta c bt ng
thc
a
a 2 + 8bc

b
b 2 + 8ca

c
c 2 + 8ab

ph cc cn thc, ta t:
x=

a
a + 8bc
2

,y=

b
b + 8ca
2

,z=

c
c + 8ab
2

R rng x, y, z (0, 1). Ta cn chng minh rng x + y + z 1. Ch rng


a2
x2
b2
y2
c2
z2
1
x2
y2
z2
=
,
=
,
=

=
.
.
8bc 1 x 2 8ca 1 y 2 8ab 1 z 2
512 1 x 2 1 y 2 1 z 2

Nh vy, ta cn chng minh rng


x + y + z 1, trong x, y, z (0, 1) v (1-x2)(1-y2)(1-z2) = 512x2y2z2
Nhng nu x + y + z < 1 th theo bt ng thc AM-GM ta c
(1-x2)(1-y2)(1-z2) > ((x+y+z)2-x2)((x+y+z)2-y2)((x+y+z)2-z2)
= (y+z)(y+z+2x)(z+x)(z+x+2y)(x+y)(x+y+2z)
2(yz)1/2.4(yzx2)1/4.2(zx)1/2.4(zxy2)1/4.2(xy)1/2.4(xyz2)1/4 = 512x2y2z2.
Mu thun.
V d 3. Cho a, b, c, d l cc s thc khng m c tng bng 4. t
Fk = (1+ak)(1+bk)(1+ck)(1+dk)
Chng minh rng F4 F3.

8 | Trn Nam Dng 6/2010

Vietnamese IMO Team Training Camp 2010


Gi s ngc li, tn ti b bn s (a, b, c, d) tha mn: a, b, c, d 0, a + b + c + d = 4 v
F4 < F3 (1).
Theo bt ng thc Cauchy-Schwarz, ta c F4F2 F32, F3F1 F22, F2F0 F12 (2). T (1)
v (2) suy ra F4 < F3 < F2 < F1 < F0 = 16 (3). T (3) ta c F4 < 16, suy ra max(a,b,c,d) <
2.
dn ti mu thun vi (3), ta s chng minh F3 F1 (4). Phn ny chng minh bng
dn bin v c xem nh mt bi tp.
V d 4. (Cezar Lupu) Cho a, b, c l cc s thc dng tha mn iu kin a + b + c +
abc = 4. Chng minh rng
a
b+c

b
c+a

c
a+b

2
.(a + b + c)
2

Gii.
Theo bt ng thc Cauchy Schwarz, ta c

(a

a
b
c
b + c + b c + a + c a + b
+
+
(a + b + c) 2
c+a
a+b
b+c

Tip tc p dng bt ng thc Cauchy Schwarz


(a + b + c)(a (b + c) + b(c + a ) + c(a + b)) a b + c + b c + a + c a + b

T suy ra
a
b+c

b
c+a

c
a+b

2
a+b+c
.(a + b + c)
2
ab + bc + ca

Nh vy ta ch cn cn chng minh a + b + c ab + bc + ca.


Bt ng thc Schur vi r = 1 c th vit di dng
9abc
4(ab + bc + ca) (a + b + c) 2
a+b+c

By gi gi s ngc li, ta c a + b + c < ab + bc + ca th


9abc
4(ab + bc + ca) (a + b + c) 2 > (a + b + c)(4 (a + b + c)) = abc(a + b + c)
a+b+c

Suy ra a + b + c < 3. Nhng khi abc < 1 v suy ra 4 = a + b + c + abc < 4, mu thun.
Bi tp
14. (MOP) Cho n 2 c nh. Cho x1, , xn l cc s dng tha mn iu kin

1
1
1
+
+ ... +
x1 x 2
xn
1
1
1
Chng minh rng
+
+ ... +
1.
n 1 + x1 n 1 + x 2
n 1 + xn
x1 + x 2 + ... + x n =

9 | Trn Nam Dng 6/2010

Vietnamese IMO Team Training Camp 2010

15. (Pu-Ro Loh) Cho a, b, c > 1 tha mn iu kin

1
1
1
+ 2
+ 2
= 1 . Chng minh rng
a 1 b 1 c 1
2

1
1
1
+
+
1.
a +1 b +1 c +1
16. Cho a, b, c l cc s thc dng tha mn iu kin

1 1 1
+ + a + b + c . Chng minh rng
a b c

a + b + c 3abc.

17. (IMO 1991) Cho tam gic ABC v im P nm trong tam gic. Chng minh rng mt trong

cc gc PAB, PBC, PCA nh hn hoc bng 300.

Mt s nh l v tnh cht chng minh bng phng php phn chng


Cui cng, ta s dng phng php phn chng chng minh mt s tnh cht quan
trng trong chng trnh ton Olympic.
nh l.
a) Nu p l s nguyn t dng 4k+1 th tn ti x sao cho x2 + 1 chia ht cho p;
b) Nu p l s nguyn t dng 4k+3 th khng tn ti x sao cho x2 + 1 chia ht cho p.
c) Nu p l s nguyn t dng 6k+1 th tn ti x sao cho x2 + 3 chia ht cho p;
d) Nu p l s nguyn t dng 6k+5 th khng tn ti x sao cho x2 + 3 chia ht cho p.
Chng minh
a) Gi s ngc li, khng tn ti x sao cho x2 + 1 chia ht cho p. Xt a bt k thuc
A = {1, 2, , p-1}. D dng chng minh c rng tn ti duy nht m(a) thuc A
sao cho a.m(a) -1 (mod p). Hn na, nu a b th m(a) m(b). Cui cng, do
khng tn ti x x2 + 1 chia ht cho p nn a m(a). Nh vy cc s 1, 2, , p-1
c phn thnh (p-1)/2 cp (a, b) vi a.b -1 (mod p). Nhn cc ng d thc
ny li vi nhau, ch k (p-1)/2 = 2k, ta c
(p-1)! (-1)2k 1 (mod p)
iu ny mu thun vi nh l Wilson: (p-1)! -1 (mod p)!
b) Gi s tn ti x sao cho x2 + 1 0 (mod p)
 x2 -1 (mod p)
 (x2)2k+1 -1 (mod p)  x4k+2 -1 (mod p)
Mt khc, theo nh l nh Fermat, ta c
x4k+2 1 (mod p)
T y suy ra 2 0 (mod p), mu thun. Vy iu gi s l sai, tc l khng tn ti x
sap cho x2 + 1 chia ht cho p.

10 | Trn Nam Dng 6/2010

Vietnamese IMO Team Training Camp 2010


c) d) c chng minh tng t da vo dy mnh tng ng sau
Phng trnh ng d x2 + 3 0 (mod p) c nghim
 Phng trnh x2 + x + 1 0 (mod p) c nghim
 Phng trnh x3 1 (mod p) c nghim x 1 (mod p).
nh l.
Nu f: R  R l mt hm cng tnh nhng khng tuyn tnh, th th G(f) = (x,
f(x)) tr mt trong R2.
C ngha l nu f(x+y) = f(x) + f(y) vi mi x, y thuc R v khng tn ti a thuc R
sao cho f(x) = ax th G(f) tr mt trong R2.
Chng minh. Gi s f l mt hm cng tnh nhng khng tuyn tnh. Ta t c = f(1) v
chn s thc sao cho f() c. Ta xt hm s g mi
g ( x) =

f ( x) cx
.
f ( ) c

Tnh cng tnh ca f suy ra g cng cng tnh trn R. Hn na g(1) = 0. S dng tnh cng
tnh ca g, ta suy ra rng g(q) = qg(1) vi mi q hu t. Nh vy ta c g(q) = 0 vi mi q
hu t.
Xt mt a Dr(x, y) bt k. Chn s hu t q sao cho |q-y| < r/2 v s hu t p sao cho |p
(x-q)| < r/2. Khi ta c
(p + q -x)2 + (q-y)2 < r2/4 + r2/4 = r2/2 < r2.
Nh vy im (p + q, q) nm trong a Dr(x, y). Hn na, theo tnh cng tnh ca g, ta
c
g(p+q) = g(p) + qg() = qg() = q
Suy ra im (p + q, q) nm trn G(g), th ca g.
iu ny chng t rng mi a m trong R2 u cha mt im no ca g. Ta v nh
vy G(g) l tr mt trong R2. Ta quay tr li vi f v s s dng thng tin ny. Ta c
f(x) = ug(x) + cx,
trong u = f() - c. Xt a Dr(a, b) bt k trong R2. Xt a D c cho bi
D = Ds(a, (b-c)/u),
vi s =

r2
, = max{2u 2 ,1 + 2c 2 } .
2

V G(g) tr mt trong R2, ta tm c s thc y sao cho (y, g(y)) thuc D. By gi xt


im (y, ug(y) + cy) thuc G(f), php kim tra trc tip cho thy im ny thuc Dr(a, b)
. iu ny chng t rng G(f) tr mt trong R2.
nh l.
Cho f, g, h l cc a thc thuc R[x] tho mn cc iu kin
i)
deg(f) = deg(g) + deg(h)
11 | Trn Nam Dng 6/2010

Vietnamese IMO Team Training Camp 2010


deg(g) > deg(h) hoc deg(g) = deg(h) v g* + h* 0, trong g*, h*
tng ng l cc h s cao nht ca g v h.
Khi vi mi n nguyn dng, tn ti khng qu 1 a thc P(x) c bc n tho mn
iu kin
P(f) = P(g)P(h).
ii)

Chng minh:
Gi s P l a thc bc n tho mn phng trnh (1), deg(f) = f, deg(g) = g, deg(h) = h,
cc h s cao nht ca P, f, g, h tng ng l P*, f*, g*, h*. So snh h s cao nht hai v
ca cc a thc trong phng trnh
P(f(x))P(g(x)) = P(h(x))
ta c P*(f*)n.P*(g*)n = P*(h*)n t suy ra P* = (h*/f*g*)n.
Nh vy, nu gi s ngc li, tn ti mt a thc Q bc n (khc P) cng tho mn
phng trnh (1) th Q* = P* v ta c
Q(x) = P(x) + R(x) vi 0 r = deg(R) < n
(ta quy c bc ca a thc ng nht 0 bng -, do deg(R) 0 ng ngha R khng
ng nht 0)
Thay vo phng trnh (1), ta c
(P(f) + R(f))(P(g) + R(g)) = P(h) + R(h)

P(f)P(g) + P(f)R(g) + R(f)P(g) + R(f)R(g) = P(h) + R(h)

P(f)R(g) + R(f)P(g) + R(f)R(g) = R(h)
(2)
By gi ta xt cc trng hp
i)
deg(f) deg(g). Gi s f > g. Khi bc ca cc a thc v tri (2) ln lt
l nf + rg, rf + ng, rf + rg, v do nf + rg > rf + ng > rf + rg nn v tri c bc l
nf + rg. Trong khi v phi c bc l rh = r(f+g) < nf + rg. Mu thun.
ii)
deg(f) = deg(g). Khi , hai a thc u tin v tri ca (2) cng c bc l nf
+ rg = ng + rf v c th xy ra s trit tiu khi thc hin php cng. Tuy nhin,
xt h s cao nht ca hai a thc ny, ta c h s ca xnf + rg trong a thc th
nht v th hai ln lt bng P*(f*)nR*(g*)r, R*(f*)rP*(g*)n. Nh th, bc ca
xnf+rg trong tng hai a thc bng
P*R*f*rg*r(f*(n-r)+g*(n-r)) 0 do f* + g* 0. Nh vy, bc ca v tri ca (2)
vn l nf + rg, trong khi bc ca v phi l rh = rf + rg < nf + rg. Mu thun.
nh l c chng minh hon ton.
Bi tp
18. Chng minh rng cc phng trnh sau y khng c nghim nguyn dng
a) 4xy x y = z2;
b) x2 y3 = 7.
19. Chng minh rng khng tn ti hm s f: N*  N* tho mn cc iu kin:
a) f(2) = 3;

12 | Trn Nam Dng 6/2010

Vietnamese IMO Team Training Camp 2010


b) f(mn) = f(m)f(n) vi mi m, n thuc N*;
c) f(m) < f(n) vi mi m < n.
20. Hi c tn ti hay khng cc s nguyn x, y, u, v, t tha mn iu kin sau
x2 + y2 = (x+1)2 + u2 = (x+2)2 + v2 = (x+3)2 + t2.
21. Chng minh nh l sau: Cho f, g, h l cc a thc khng hng tha mn iu kin deg(f) + deg(g) =
deg(h), Q l mt a thc cho trc. Khi , vi mi s nguyn dng n v s thc a, tn ti nhiu nht
mt a thc P tha mn ng thi cc iu kin sau: i) deg(P) = n, ii) P* = a iii) P(f)P(g) = P(h) + Q.

2. Quy np ton hc
Quy np ton hc l mt trong nhng nt c trng ca suy lun trong ton hc. T duy
quy np rt cn thit trong s hc, i s, t hp, hnh hc v gii tch, ni chung l trong
tt c cc lnh vc ca ton hc.
Quy np ton hc v bt ng thc
Gp cc bt ng thc c nhiu bin s, ta c th ngh ngay n php quy np ton hc.
D nhin, vic p dng quy np th no lun l c mt ngh thut.
V d 1. (Chng minh bt ng thc Cauchy bng quy np tin).
Cho a1, a2, , an l cc s thc khng m. Chng minh rng ta lun c
a1 + a 2 + ... + a n n n a1 a 2 ...a n

Trong cc ti liu, bt ng thc ny thng c chng minh bng php quy np li,
hay quy np kiu Cauchy. y chng ta trnh by mt php chng minh khc.
C s quy np vi n = 1, 2 c kim tra d dng. Gi s bt ng thc c chng
minh cho n s. Xt n+1 s khng m a1, a2, , an+1. t a1a2an+1 = An+1. Nu tt c cc
s bng nhau th bt ng thc ng. Trong trng hp ngc li, phi tn ti hai s ai, aj
sao cho ai < A < aj. Khng mt tnh tng qut, c th gi s an < A < an+1. Khi ta c (an
A)(an+1 A) < 0, suy ra an + an+1 > anan+1/A + A. T ta c
a1 + a2 + + an + an+1 > a1 + + an-1 + anan+1/A + A(1)
By gi p dng bt ng thc Cauchy cho n s a1 + + an-1 + anan+1/A ta c
a1 + a 2 + ... + a n 1 + a n n n a1a 2 ...a n 1

a n a n +1
= nA
A

Kt hp vi (1) ta c pcm.
V d 2. Cho n 2 v cho x1, x2, , xn l cc s thc thuc [0, 1]. Chng minh rng
x1(1-x2) + x2(1-x3) + + xn(1-x1) [n/2]

13 | Trn Nam Dng 6/2010

Vietnamese IMO Team Training Camp 2010


Vn bi ton ny l bc chng minh t n  n+1 trong trng hp n chn l khng
th (do lc v phi khng thay i v ta cn chng minh phn thay i v tri nh
hn hay bng 0:
xn(1-xn+1) + xn+1(1-x1) xn(1-x1) 0
<=> xn(x1-xn+1) + xn+1(1-x1) 0
R rng biu thc v tri c th nhn c nhng gi tr dng v bc quy np ca chng
ta khng thc hin c.
Ta c th vt qua c kh khn ny nu thc hin bc quy np nhy cch, tc l t n
 n+2. Khi , do [(n+2)/2] [n/2] = 1 nn ta cn chng minh:
xn(1-xn+1) + xn+1(1-xn+2) + xn+2(1-x1) xn(1-x1) 1.
iu ny tng ng vi
A = xn(x1-xn+1) + xn+1(1-xn+2) + xn+2(1-x1) 1.
Bt ng thc ny c th chng minh c kh d dng (chng ti dnh cho bn c).
Cui cng, ta cn chng minh c s quy np, trong trng hp ny l trng hp n = 2
v n = 3.
x1(1-x2) + x2(1-x1) 1
v
x1(1-x2) + x2(1-x3) + x3(1-x1) 1
Bt ng thc th nht ng do
x1(1-x2) + x2(1-x1) = 1 (1x1)(1x2) x1x2 1
Bt ng thc th hai ng do
x1(1-x2) + x2(1-x3) + x3(1-x1) = 1 (1x1)(1x2)(1x3) x1x2x3.
Ch rng, trong php chng minh bt ng thc A 1 (php chng minh quy np) c
th s dng n bt ng thc x1(1-x2) + x2(1-x3) + x3(1-x1) 1.
V d 3. Cho n 2 v x1, x2, , xn l n s nguyn phn bit. Chng minh rng
(x1-x2)2 + (x2-x3)2 + + (xn x1)2 4n 6
Ta th xt bc quy np t n  n+1. Khi v phi thay i 4 n v, trong khi thay i
v tri l
A = (xn-xn+1)2 + (xn+1-x1)2 (xn x1)2
Ta cn chng minh A 4.
Nu nhn k li bt ng thc cn chng minh v cc iu kin rng buc th ta thy rng
bt ng thc A 4 ni chung khng ng ! Vy phi lm th no?
Ta vit li bt ng thc di dng
A = xn+1(2xn+1-xn-x1) + 2x1xn
14 | Trn Nam Dng 6/2010

Vietnamese IMO Team Training Camp 2010


By gi, ta mi ch n hai tnh cht quan trng ca bt ng thc ban u
1) V tri khng thay i nu ta cng thm vo mi s hng xi mt i lng a c
nh. Do ta c th gi s xn+1 = 0
2) y l bt ng thc hon v, do ta c th gi s xn+1 =
min{x1,x2,,xn,xn+1}
T y suy ra A = 2x1xn 2.2 = 4 (v x1, x2 > 0 l cc s nguyn phn bit nn x1x2
1.2 = 2).
Bi ton c gii quyt.
V d 4. Cho cc s dng a1, a2, , an tha mn iu kin a1 + a2 + + an = n. Chng
minh rng ta c bt ng thc
1
1
1
8(n 1)
+
+ ... +
n
(1 a1 a 2 ...a n ) .
a1 a 2
an
n2

Gii.
Ta chng minh kt qu tng qut hn
1
1
1
+
+ ... +
n m n (1 a1a 2 ...a n )
a1 a 2
an
8(n 1)
vi mi mn
.
n2

Vi n = 1, bt ng thc hin nhin ng. Gi s bt ng thc ng n n = k, ta


chng minh bt ng thc cng ng vi n = k+1. Tht vy, gi s ak+1 = max{a1, a2, ,
a1 + ... + a k
1. t bi = ai/b suy ra b1 + b2 + + bk = k. Ch l
k
k +1
8k
8(k 1
kb + a k +1
k +1
k
mk +1b a k +1 mk +1b a k +1 mk +1

= mk +1 =
2
(k + 1)
k2
k +1

ak}, suy ra b =

Do , s dng gi thit quy np ta c


1
1
+ ... + k mk +1b k +1 a k +1 (1 b1 ...bk )
b1
bk

a ...a
1
1 k
+ ... +
mk +1b k a k +1 (1 1 k k )
a1
ak b
b

1
1
k
+ ... +
+ mk +1 a1 ...a k a k +1 + mk +1b k a k +1
a1
ak
b

Cui cng, ta phi chng minh

15 | Trn Nam Dng 6/2010

Vietnamese IMO Team Training Camp 2010


k
1
+ mk +1b k a k +1 +
k 1 mk +1 0
b
a k +1
k
1
+
(k + 1) mk +1 (1 b k a k +1 )
b a k +1

k
1
+
(k + 1) mk +1 (1 b k (k + 1 kb))
b k + 1 kb
k (k + 1)
mk +1
b(k + 1 kb)(1 + 2b + ... + kb k 1 )

Bt ng thc ny ng v mk +1

8k
(k + 1) 2
,
b
1
,
b
(
k
1
kb
)

.
4k
(k + 1) 2

Vy ta c iu phi chng minh.


Bi tp
1. Chng minh rng vi x1 x2 xn 0 ta c bt ng thc
n

i =1

i =1

xi2

xi
i

2. Chng minh rng nu a1, a2, , an l cc s nguyn dng phn bit th ta c bt ng thc

(a + a ) 2 ai3

i =1
i =1
n

7
i

5
i

3. (Bt ng thc Mc-Lauflin) Vi mi s thc a1, a2, , a2n v b1, b2, , b2n ta c bt ng thc
2

2n

a
b

(
a
b

a
b
)

a k bk

2 k 2 k 1
2 k 1 2 k
k =1
k =1
k =1

k =1

2n

2n

2
k

2
k

4. Cho x1, x2, , xn l cc s thc dng. Chng minh rng

xi2
n

2
2
i =1 xi + xi +1 xi + 2
n

trong xn+1 = x1, xn+2 = x2.


5. Cho a1, a2, , an l cc s thc dng tha mn iu kin a1 + a2 + + an = n. Chng minh rng
(n-1)(a12+a22 ++an2) + na1a2an n2.
6. Cho n 3 v a1, a2, , an l cc s nguyn dng tha mn iu kin bi =

ai 1 + ai +1
nguyn vi mi
ai

i = 1, 2, , n ( y an+1 = a1, a0 = an). Chng minh rng khi ta c bt ng thc


n

2n bi 3n 2
i =1

16 | Trn Nam Dng 6/2010

Vietnamese IMO Team Training Camp 2010


Quy np trong s hc
Quy np c s dng rng ri trong s hc, c bit l trong cc bi ton v ng d, v
bc theo modulo m. Di y ta xem xt mt s v d kinh in.
nh l nh Fermat: Nu p l s nguyn t th ap a chia ht cho p vi mi a nguyn.
nh l ny c th chng minh bng php quy np ton hc, s dng tnh cht C pk chia
ht cho p vi mi k = 1, 2, , p-1.
V d 4. (VMO 1997) Chng minh rng vi mi s nguyn dng n u chn c s
nguyn dng k 19k 97 chia ht cho 2n.
Vi n = 1, n = 2 ta chn k = 2 nn ch cn xt vi n 3. Ta c nhn xt sau
19 2 1 = 2 n .t n vi tn l.
(1)
Tht vy, vi n = 3, khng nh 1 ng. Gi s khng nh ng vi n. Khi
19 2 1 = (19 2 1)(19 2 + 1) = 2 s n 2 n t n = 2 n +1 ( s n t n ) vi (sntn) l.
Nhn xt c chng minh.
n2

n 1

n2

n2

Ta chng minh bi ton bng quy np. Vi n = 3 ng. Gi s tn ti kn thuc N* sao cho
19 kn 97 = 2 n.a
Nu a chn th 19 k n 97 chia ht cho 2n+1. Nu a l, t kn+1 = kn + 2n-2. Khi theo nhn

xt ta c
n2

19 k n +1 97 = 19 2 (19 k n 97) + 97(19 2

n2

n2

1) = 2 n (19 2 a + 97t n )

chia ht cho 2n+1 (pcm).


Bi tp
4. Chng minh rng vi mi s nguyn dng n s n! tho mn iu kin sau: vi mi c s ca n,
khc vi n! c th tm c mt c s khc ca n! sao cho tng hai c s li l c s ca n!.
5. Chng minh rng nu s nguyn dng N c th biu din di dng tng bnh phng ca ba s
nguyn chia ht cho 3 th n cng c th biu din di dng tng bnh phng ca ba s khng chia ht
cho 3.
6. Chng minh rng tn ti v s hp s n sao cho 3n-1 2n-1 chia ht cho n.

Quy np trong cc bi ton tr chi


Cc bi ton tr chi chnh l dng ton s dng n quy np ton hc nhiu nht. Ch
l quy np ton hc y bao gm hai phn: d on cng thc v chng minh cng
thc v trong rt nhiu trng hp, vic d on cng thc ng vai tr then cht.
17 | Trn Nam Dng 6/2010

Vietnamese IMO Team Training Camp 2010

V d 5. Hai ngi A v B cng chi mt tr chi. Ban u trn bn c 100 vin ko. Hai
ngi thay phin nhau bc ko, mi ln c bc k vin vi k {1, 2, 6} . Hi ai l
ngi c chin thut thng, ngi i trc hay ngi i sau?
Ta s khng bt u t 100 vin ko m bt u t nhng s ko nh hn. Gi s ban u
trn bn c n vin ko. Nu n = 1, 2, 6 th r rng ngi th nht c chin thut thng (ta
gi n gin l ngi th nht thng). Vi n = 3 th ngi th hai thng, bi ngi th
nht ch c th bc 1 hoc 2 vin v tng ng ngi th hai bc 2 hay 1 vin thng.
Vi n = 4 ngi th nht thng bng cch bc 1 vin ko v y ngi th hai vo th
thua. Tng t, vi n = 5 ngi th nht thng. Vi n = 7, ngi th hai thng v c ba
cch i c th ca ngi th nht (bc 1, 2, 6 vin) u dn n th thng cho ngi th
hai (tng ng cn 6, 5, 1 vin ko trn bn), n = 8 ngi th nht thng
Bng cch l lun tng t nh vy, ta lp c bng sau

n
KQ

1 2
1 1

3
2

4
1

5
1

6
1

7
2

8
1

9
1

10
2

11 12
1 1

13
1

14
2

15
1

16
1

17
2

T bng kt qu, c th d on c l ngi th nht s thng nu n c s d l 1, 2,


4, 5, 6 trong php chia cho 7, v ngi th hai s thng nu n c s d l 0, 3 trong php
chia cho 7.
Sau khi c d on ta tm cch chng minh cht ch d on ca mnh bng php quy
np ton hc. t n = 7k+r vi r = 1, 2, , 6, 7 ta chng minh d on trn bng quy np
theo k. Vi k = 0 mnh c kim chng qua bng trn.
Xt n = 7(k+1) + r vi r = 1, 2, , 6, 7
Nu r = 1, 2, 6, ngi th nht bc tng ng 1, 2, 6 vin a v trng hp trn bn
cn 7k+7 vin ko l th thua cho ngi th hai (theo gi thit quy np), v th ngi th
nht thng.
Nu r = 3, ngi th nht c 3 cch bc
+ Bc 1 vin, cn 7(k+1) + 2 l th thng cho ngi th hai (ta va chng minh
trn)
+ Bc 2 vin, cn 7(k+1) + 1, tng t cng l th thng cho ngi th hai
+ Bc 6 vin, cn 7k + 4 vin l th thng ca ngi th hai (theo gi thit quy
np).
Nh vy trng hp ny ngi th nht thua.
Nu r = 4, 5, ngi th nht bc tng ng 1, 2 vin a v trng hp 7(k+1) + 3 l
th thua cho ngi th hai, v v vy ngi th nht thng.
Cui cng, trng hp r = 7, ngi th nht c 3 cch bc
+ Bc 1 vin, cn 7(k+1) + 6 l th thng cho ngi th hai (chng minh trn)
+ Bc 2 vin, cn 7(k+1) + 5 l th thng cho ngi th hai (chng minh trn)
18 | Trn Nam Dng 6/2010

Vietnamese IMO Team Training Camp 2010


+ Bc 6 vin, cn 7(k+1) + 1 l th thng cho ngi th hai (chng minh trn)
Vy ngi th nht thua.
Nh th d on ca chng ta c chng minh hon ton.
V 100 chia 7 d 2 nn theo l lun trn th ngi th nht c chin thut thng.
V d 6. Cu b v Freken Bock cng chi mt tr chi. Trn bn c mt s ko. Bc i
u tin, cu b chia s ko thnh 3 ng khc rng, sau Freken chn ra 2 ng a
cho Carlson, ng cn li Freken li chia ra thnh 3 ng khc rng v cu b li chn
ra hai ng a cho Carlson, ng cn li chia thnh 3 ng khc rng Ai n lt
mnh khng i c na th thua. Hi ai l ngi c chin thut thng nu trn bn c:
a) 7 vin ko ;
b) 9 vin ko ;
c) 12 vin ko ;
d) 14 vin ko ;
e) Mt s ko bt k.
Bi tp
7. a) Trn bng c s 2010. Hai ngi A v B cng lun phin thc hin tr chi sau: Mi ln thc hin,
cho php xo i s N ang c trn bng v thay bng N-1 hoc [N/2]. Ai thu c s 0 trc l thng
cuc. Hi ai l ngi c chin thut thng, ngi i trc hay ngi i sau.
b) Cng cu hi vi lut chi thay i nh sau: Mi ln thc hin, cho php xo i s N ang c trn
bng v thay bng N-1 hoc [(N+1)/2].
8. C bng ch nht gm m x n . Hai ngi A v B cng lun phin nhau t mu cc ca bng, mi ln
t cc to thnh mt hnh ch nht. Khng c php t nhng t. Ai phi t cui cng l thua.
Hi ai l ngi c chin thut thng, ngi i trc hay ngi i sau?
9. An v Bnh chi tr on s. An ngh ra mt s no nm trong tp hp X = {1, 2, , 144}. Bnh c
th chn ra mt tp con bt k A ca X v hi S ca bn ngh c nm trong A hay khng ? . An s tr
li C hoc Khng theo ng s tht. Nu An tr li c th Bnh phi tr cho An 2.000 ng, nu An tr
li Khng th Bnh phi tr cho An 1.000 ng. Hi Bnh phi tt t nht bao nhiu tin chc chn tm
ra c s m An ngh ?

Quy np trong bi ton m


Xy dng cng thc truy hi l mt trong nhng phng php quan trng gii bi ton
m. T tng quy np y rt r rng: tm cng thc cho bi ton m vi kch
thc n, ta s dng kt qu ca bi ton m tng t vi kch thc nh hn.
V d 7. (Bi ton chia ko ca Euler)
Cho k, n l cc s nguyn dng. Tm s nghim nguyn khng m ca phng trnh
x1 + x2 + + xn = k
(*)
19 | Trn Nam Dng 6/2010

Vietnamese IMO Team Training Camp 2010

Gii. Gi s nghim nguyn khng m ca phng trnh trn l S(n, k). D dng thy
rng S(1, k) = 1. tnh S(n, k), ta ch rng (*) tng ng vi
x1 + ...+ xn-1 = k - xn (**)
Suy ra vi xn c nh th s nghim ca (**) l S(n-1, k-xn). T ta c cng thc
S(n, k) = S(n-1, k) + S(n-1, k-1) + ...+ S(n-1, 0)
y c th coi l cng thc truy hi tnh S(n, k). Tuy nhin, cng thc ny cha tht tin
li. Vit cng thc trn cho (n, k-1) ta c
S(n, k-1) = S(n-1, k-1) + S(n-1, k-2) + ...+ S(n-1, 0)
T y, tr cc ng thc trn v theo v, ta c
S(n, k) - S(n, k-1) = S(n-1, k)
Hay S(n, k) = S(n, k-1) + S(n-1, k)
T cng thc ny, bng quy np ta c th chng minh c rng S(n, k) = Ckn+k-1.
Trong nhiu trng hp, vic xt thm cc bi ton ph s gip chng ta thit lp nn cc
h phng trnh truy hi, t suy ra cng thc truy hi cho cc bi ton chnh.
V d 8. Xt tp hp E = {1, 2, , 2010}. Vi tp con A khc rng ca E, ta t
r(A) = a1 a2 + + (-1)k-1ak
trong a1, a2, , ak l tt c cc phn t ca A xp theo th t gim dn. Hy tnh tng
S = r ( A) .
A E

t En = {1, 2, , n} v S n =

r ( A) . Xt Sn+1, bng cch chia cc tp con ca En+1

A E n

thnh 2 loi, loi khng cha n+1 v cha n+1, ta c


S n+1 =

r ( A) = r ( A) + r ( A {n + 1}) = r ( A) + (n + 1 r ( A)) = (n + 1)2 .


n

A En +1

A E n

A E n

A E n

A E n

Ghi ch. y l tnh hung may mn c bit khi chng ta truy hi m khng truy hi,
ngha l ra c cng thc tng minh lun.
V d 9. C 2n ngi xp thnh 2 hng dc. Hi c bao nhiu cch chn ra mt s ngi
(t nht 1) t 2n ngi ny, sao cho khng c hai ngi no ng k nhau c chn.
Hai ngi ng k nhau l hai ngi c s th t lin tip trong mt hng dc hoc c
cng s th t hai hng.
Gi Sn l s cch chn ra mt s ngi t 2n ngi xp thnh 2 hng dc v Tn l s cch
chn ra mt s ngi t 2n-1 ngi xp thnh 2 hng dc, trong khuyt mt ch u
ca mt hng. Ta c S1 = 2, T1 = 1.
1 3
2 4
Hnh 1. Sn vi n = 5
20 | Trn Nam Dng 6/2010

Vietnamese IMO Team Training Camp 2010

1 2

Hnh 2. Tn vi n = 5
Xt 2n ngi xp thnh 2 hng dc (nh hnh 1). Ta xt cc cch chn tho mn iu
kin u bi. Xy ra cc kh nng sau :
1) Ngi v tr s 1 c chn : Khi ngi v tr s 2 v s 3 khng c
chn  C Tn-1 + 1 cch chn (+1 l do b sung cch chn khng chn g
c )
2) Ngi v tr s 2 c chn : Tng t, c Tn-1 + 1 cch chn.
3) C hai ngi v tr s 1 v s 2 u khng c chn: C Sn-1 cch chn.
Vy ta c Sn = Sn-1 + 2Tn-1+ 2 (1).
Xt 2n-1 ngi xp thnh 2 hng dc (nh hnh 2). Ta xt cc cch chn tho mn iu
kin u bi. Xy ra cc kh nng sau :
1) Ngi v tr s 1 c chn : Khi ngi v tr s 2 khng c chn 
c Tn-1 + 1 cch chn
2) Ngi v tr s 1 khng c chn : c Sn-1 cch chn.
Vy ta c Tn = Sn-1 + Tn-1 + 1 (2)
T (1) ta suy ra 2Tn-1 = Sn Sn-1 2, 2Tn = Sn+1 Sn 2. Thay vo (2), ta c
Sn+1 Sn 2 = 2Sn-1+ Sn Sn-1 2 + 2
Sn+1 = 2Sn + Sn-1 + 2
T y d dng tm c
Sn =

(1 + 2 ) n +1 + (1 2 ) n +1 2
2

Bi tp
10. Tm s cch lt ng i kch thc 3 x 2n bng cc vin gch kch thc 1 x 2.
11. Tm s tt c cc b n s (x1, x2, , xn) sao cho
(i) xi = 1 vi i = 1, 2, , n.
(ii) 0 x1 + x2 + + xr < 4 vi r = 1, 2, , n-1 ;
(iii) x1 + x2 + + xn = 4.
12. Trn bn c 365 tm ba m trn mt p xung ca n c ghi cc s khc nhau. Vi 1.000 ng An c
th chn ba tm ba v yu cu Bnh sp xp chng t tri sang phi sao cho cc s vit trn chng c
xp theo th t tng dn. Hi An, b ra 2.000.000 c th chc chn sp xp 365 tm ba sao cho cc s
c vit trn chng c xp theo th t tng dn hay khng ?
13. (Bi ton con ch, IMO 1979) Gi A v E l hai nh i din ca mt bt gic. T mt nh bt k
ngoi tr E, con ch nhy n hai nh k. Khi n nhy n nh E th n ngng li. Gi an l s cc
ng i khc nhau vi ng n bc nhy v kt thc ti E. Chng minh rng a2n-1 = 0,

a2n =

(2 + 2 ) n 1 (2 2 ) n 1

21 | Trn Nam Dng 6/2010

Vietnamese IMO Team Training Camp 2010


Quy np v mt s nh l trong ti u t hp
nh l 1. (Hall, 1935) Cho th hai phe X, Y. Vi mi tp con A thuc X, gi G(A) l
tp cc nh thuc Y k vi mt nh no thuc A. Khi iu kin cn v tn
ti mt n nh f: X  Y sao cho x k f(x) l |G(A)| |A| vi mi A khc rng thuc X.
Chng minh. iu kin cn l hin nhin: Nu tn ti n nh f th vi mi A = {x1, x2,
, xr} thuc X, ta c G(A) cha cc phn t phn bit f(x1), , f(xr), do |G(A)| r =
|A|.
Ta chng minh iu kin bng quy np theo |X|. Khi |X| = 1, khng nh l hin nhin.
Gi s nh l ng vi cc tp X vi |X| < n. Gi s by gi |X| = n. Ta xt hai trng
hp:
1) Gi s vi mi A X (A X), ta c |G(A)| > |A|. Chn mt phn t x0 bt k thuc X,
theo iu kin |G({x0})| 1, do tn ti y0 thuc Y k vi X. Ta t f(x0) = y0. By gi
xt X = X \{x} v Y = Y \ {y}, A X v G(A) l tp cc nh thuc Y k vi A. Khi
|G(A)| |G(A)| - 1 |A|. V |X| < |X| nn theo gi thit quy np, tn ti n nh f: X
 Y sao cho f(x) k x vi mi x thuc x. B sung thm f(x0) = y0 ta c n nh f: X
 Y tha mn yu cu nh l.
2) Trong trng hp ngc li, tn ti A X (A X) sao cho |G(A)| = |A|. Khi , do |A|
< |X| nn tn ti n nh f: A  G(A). Xt X = X \ A, Y = Y \ G(A). Xt B thuc X v
G(B) l tp cc nh thuc Y k vi B. Nu |G(B)| < |B| th ta c
|G(A B)| = |G(A)| + |G(B)| < |A| + |B| = |A B|
mu thun vi iu kin nh l. Nh vy ta c |G(B)| |B| vi mi B thuc X. Theo gi
thit quy np, tn ti n nh g: X  Y sao cho g(x) k vi x. Nh vy, ta c th xy
dng c n nh h: X  Y sao cho h(x) k vi x: c th h(x) = f(x) nu x thuc A v
h(x) = g(x) nu x thuc X \ A.
Quan h trn tp hp X c gi l mt quan h th t nu tha mn ng thi cc
iu kin sau:
i)
x x vi mi x thuc X (tnh phn x)
ii)
Nu x y, y x th x = y (tnh phn xng)
iii)
Nu x y, y z th x z (tnh bc cu)
Mt tp hp m trn xc nh mt quan h th t c gi l mt tp sp th t.
Cho X l mt tp sp th t, hai phn t x v y thuc X c gi l so snh c nu x
y hoc y x. Trong trng hp ngc li, ta ni x v y khng so snh c.
Mt tp con C ca X c gi l mt xch nu hai phn t bt k thuc C u so snh
c. Mt tp con A ca X c gi l mt i xch nu hai phn t bt k thuc A u
khng so snh c.

22 | Trn Nam Dng 6/2010

Vietnamese IMO Team Training Camp 2010


Phn t x thuc X c gi l phn t cc i nu t x y suy ra y = x. Phn t x c
gi l cc tiu nu t y x suy ra y = x. Phn t x thuc X c gi l ln nht nu x y
vi mi y thuc X v c gi l nh nht nu x y vi mi y thuc X. Xch C c gi
l cc i nu nh khng tn ti mt xch C cha C vi |C| > |C|. Tng t ta nh
ngha i xch cc i.
nh l 2. (Dilworth 1950) Cho mt tp sp th t X. S phn t ln nht ca mt i
xch ca X bng s nh nht cc xch ri nhau hp thnh X.
Chng minh 1. Gi M = max{|A| | A l i xch} v m l s nh nht cc xch ri nhau
hp thnh X. Nh vy tn ti i xch A ca X cha M phn t. V mt xch ch cha
c nhiu nht 1 phn t ca 1 i xch nn r rng ta c m M.
Ta chng minh m M bng quy np theo |X|. Gi a l mt phn t cc i ca X v M l
kch thc ca i xch ln nht trong X = X \ {a}. Khi , theo gi thit quy np X l
hp ca M xch ri nhau C1, C2, , CM. Ta cn chng minh rng hoc X cha i xch
vi M+1 phn t, hoc X l hp ca M xch. By gi, mi i xch kch thc M (M-i
xch) trong X cha mt phn t t mi Ci. Gi ai l phn t ln nht trong Ci thuc vo
mt M-i xch no trong X. D dng thy rng A = {a1, a2, , aM} l mt i xch
(nu chng hn ai < aj th v aj thuc vo mt M-i xch no v i xch ny li cha
mt phn t bi ca Ci nn theo tnh ln nht ca ai, ta c bi ai < aj iu ny mu thun v
bi v aj cng thuc mt i xch). Nu A {a} l mt i xch trong X th ta c pcm.
Trong trng hp ngc li, ta c a > ai vi i no . Khi K = {a} {x Ci : x ai}
l mt xch trong X v khng c M-i xch trong X \ K (v ai l phn t ln nht ca Ci
tham gia trong cc i xch nh vy), v th X \ K l hp ca M-1 xch.
Chng minh 2. (Theo H. Tverberg 1967)
 Hin nhin ta c m M.
 Ta chng minh M m bng quy np theo |X|.
 iu ny l hin nhin nu |X|=0.
 Gi s C l xch cc i trong X.
 Nu mi i xch trong X\C c nhiu nht M-1 phn t th xong.
 Gi s {a1,, aM} l mt i xch trong P\C.
 nh ngha S- = {x X: i [ x ai]}, S+ {x X: i [ ai x]}
 V C l cc i, phn t ln nht ca C khng nm trong S- .
 Theo gi thit quy np, nh l ng vi S- .
 V th, S- l hp ca M xch ri nhau S-1, , S-M, trong ai S-i
.
 Gi s rng x S i v x > ai. Nu nh tn ti aj vi x aj, ta s c ai < x
aj. Mu thun. V vy ai l phn t ln nht trong S-i , i=1,,M.
 Lm tng t i vi S+i, ta c ai l phn t nh nht trong S+i.
 Kt hp cc xch li ta c iu phi chng minh.
23 | Trn Nam Dng 6/2010

Vietnamese IMO Team Training Camp 2010


3. Nguyn l Dirichlet
Nguyn l Dirichlet dng c in thng c dng chng minh tn ti theo kiu
khng xy dng (non-constructive), tc l bit i tng tn ti nhng khng ch ra c
th.
Nguyn l Dirichlet trong s hc
Trong s hc, nguyn l Dirichlet thng lin quan n cc bi ton chia ht, nguyn t
cng nhau. V d cc bi ton kinh in sau.
V d 1. Chn ra n+1 s t 2n s nguyn dng u tin.
a) Chng minh rng trong cc s c chn, c hai s phn bit x, y nguyn t
cng nhau.
b) Chng minh rng trong cc s c chn, c hai s x > y m x chia ht cho y.
V d 2. Chng minh rng t n s nguyn bt k lun c th chn ra mt s hoc mt s
s c tng chia ht cho n.
V d 3. (nh l Fermat-Euler v tng hai bnh phng)
Chng minh rng nu p l s nguyn t dng 4k+1 th tn ti cc s nguyn a, b sao cho
p = a2 + b2.
Chng minh. V p c dng 4k+1 nn theo kt qu ca nh l phn u, tn ti s
nguyn N sao cho N2 + 1 chia ht cho p, hay ni cch khc, N2 -1 (mod p). Xt cc s
dng x + Ny vi x, y l cc s nguyn thuc [0, [ p ]] . C tt c ([ p ] + 1) 2 s nh vy.
V ([ p ] + 1) 2 > p nn theo nguyn l Dirichlet, tn ti hai cp s (x, y) (x, y) sao cho
x + Ny x + Ny (mod p). T y suy ra
x x N(y y) (mod p)
=>
(x x)2 N2(y y)2 (mod p)
By gi, nh li rng N2 - 1 (mod p), ta suy ra
(x x)2 - (y y)2 (mod p)

(x x)2 + (y y)2 (mod p)
Cui cng, ch rng 0 < (x x)2 + (y y)2 < 2p ta suy ra iu phi chng minh.
Ngoi k thut kinh in vi chung v th, ta c th s dng mt bin th ca nguyn l
Dirichlet nh sau:
Tnh cht. Nu A, B l cc tp hp tho mn iu kin |A| + |B| > |A B| th
0.
Sau y l mt p dng ca tnh cht ny.
24 | Trn Nam Dng 6/2010

AB

Vietnamese IMO Team Training Camp 2010

V d 4. Chng minh rng nu p l s nguyn t dng 4k+3 th tn ti cc s nguyn x, y


sao cho x2 + y2 + 1 chia ht cho p.
Chng minh. t ri = i2 mod p vi i = 1, 2, , (p-1)/2 v si = 1 i2 mod p, i = 1, 2, ,
(p-1)/2 th d dng chng minh c rng ri i mt phn bit v si i mt phn bit.
Hn na, ri v si u thuc {1, 2, , p-1}.
t A = {r1, , r(p-1)/2}, B = {s1, , s(p-1)/2} th |A| = |B| = (p-1)/2 v |A B| p-1. Xy
ra hai trng hp
Trng hp 1. Nu | A B | < p-1 th theo tnh cht nn trn, ta c A B , tc l tn
ti i, j sao cho ri = sj, tng ng vi i2 - 1- j2 (mod p)  i2 + j2 + 1 chia ht cho p
(pcm).
Trng hp 2. Nu | A B | = p-1 th A B = v nh vy, cc s r1, r2, , r(p-1)/2, s1,
s2, , s(p-1)/2 i mt phn bit v ta c
r1 + r2 + + r(p-1)/2 + s1 + s2 + + s(p-1)/2 = 1 + 2 + + p-1 0 (mod p)
iu ny mu thun v theo nh ngha ca ri v si, ta c
r1 + r2 + + r(p-1)/2 + s1 + s2 + + s(p-1)/2 12 + 22 + + [(p-1)/2]2 + (-1-12) + +
(-1 [(p-1)/2]2) -(p-1)/2 (mod p).
Vy trng hp 2 khng xy ra, v nh th ta ri vo trng hp 1. Ta c iu phi
chng minh.
Ghi ch. L lun A v B v khng B suy ra A c gi l Tam on lun ri.
Bi tp
1. Xt dy s Fibonacci xc nh bi F1 = F2 = 1, Fn+1 = Fn + Fn-1 vi mi n 2. Chng minh rng vi mi
s nguyn dng m > 1. Tn ti v s s hng ca dy s chia ht cho m.
2. T khong (22n, 23n) chn ra 22n-1+1 s l. Chng minh rng trong cc s c chn, tn ti hai s m
bnh phng mi s khng chia ht cho s cn li.
3. a) Chng minh rng khng tn ti s nguyn dng n sao cho 10n + 1 chia ht cho 2003.
b) Chng minh rng tn ti cc s nguyn dng m, n sao cho 10m + 10n + 1 chia ht cho 2003.
4. (Vietnam TST 2001) Dy s nguyn dng a1, a2, , an, tho mn iu kin
1 an+1 an 2001 vi mi n = 1, 2, 3, Chng minh rng tn ti v s cp s p, q sao cho q > p v aq
chia ht cho ap.

25 | Trn Nam Dng 6/2010

Vietnamese IMO Team Training Camp 2010


Nguyn l Dirichlet trong i s
Trong i s nguyn l Dirichlet c th hin qua tnh cht c bn sau: Nu trn on
[a, b] c n s thc x1, x2, , xn (n 2) th tn ti cc ch s i j sao cho
|xi-xj| (ba)/(n-1).
V d 5. Gia 7 s thc bt k lun tm c 2 s x v y sao cho 0 <

x y
1

. . Gi
1 + xy
3

cc s cho l a1, a2, , a7. Vi mi s thc a, tn ti s thuc khong (-/2, /2)


sao cho a = tg(). Gi s a1 = tg(1), a2 = tg(2), , a7 = tg(7). Theo tnh cht nu trn,
trong 7 s 1, 2, , 7 tn ti hai s c hiu khng vt qu /6. Gi s hai s ny l
v , trong > . Khi
tg ( ) tg ( )
1

0<
= tg ( ) tg =
.
1 + tg ( )tg ( )
6
3
Nh vy cc s x = tg() v y = tg() l cc s cn tm.
nh l Kronecker v s tr mt l mt nh l c nhiu ng dng trong gii tch, i s,
gii tch phc. Di y ta xt chng minh rt s cp ca nh l ny ( dng tng
ng)
nh l Kronecker. Nu l s v t th tp hp S ={ {n} | n N*} tr mt trong [0,
1].
Chng minh. Ta cn chng minh rng vi mi khong (a, b) [0, 1], tn ti s nguyn
dng n sao cho {n} (a, b).
Trc ht, ta tm s nguyn dng N sao cho N > 1/(b-a). By gi xt N+1 s {},
{2}, , {(N+1)} thuc on [0, 1]. Theo nguyn l Dirichlet, tn ti hai s {p},
{q} vi 1 p < q N+1 sao cho |{p} {q}| 1/N.
rng {q} {p} = q - [q] - p + [p] = (q-p) - [q] + [p]. Do nu {q}
{p} > 0 th {(q-p)} = {q} {p}, cn nu {q} {p} < 0 th
{(q-p)} = 1
({q} {p})
Ta xt hai trng hp
Trng hp 1. 1/N > {q} {p} > 0. Khi t k = q p, ta tm c s nguyn
dng k sao cho 0 < {k} < 1/N.
By gi ta gn m l s nguyn dng nh nht sao cho m{k} > a th ta c
(m1){k} a. Do m{k} a + {k} < a + 1/N < b. Suy ra m{k} (a, b) [0, 1].

26 | Trn Nam Dng 6/2010

Vietnamese IMO Team Training Camp 2010


Nhng mk = m[k] + m{k}. Do 0 < m{k} < 1 nn t y ta c {mk} = m{k}.
t n = mk, ta c {n} (a, b) (pcm).
Trng hp 2. 0 > {q} {p} > -1/N. Khi t k = q p, ta tm c s nguyn
dng k sao cho 1 1/N < {k} < 1. t = 1 - {k} th 0 < < 1/N. Chng minh
tng t nh trn, ta tm c s nguyn dng m sao cho m thuc (1-b, 1-b), tc l
1 b < m(1 - {k}) < 1 a

a < m{k} m + 1 < b

a < {m{k}} < b
Cui cng, ta c mk = m[k] + m{k} nn {mk} = {m{k}} nn t n = mk ta c
iu phi chng minh.
Mt tnh hung rt n gin khc ca nguyn l Diriclet li c nhng ng dng rt hiu
qu trong nhiu bi ton chng minh bt ng thc, c bit l cc bt ng thc c iu
kin. l ch sau: Vi m l mt s thc cho trc v n 3 s thc a1, a2, , an bt k
th lun tm c hai s trong cc s ny nm cng mt pha i vi m.
Gi hai s l x v y th ta c bt ng thc hin nhin sau: (x-m)(y-m) 0, t xy +
m2 m(x+y). Nh vy, ta so snh c hai i lng khng cng bc vi nhau. Sau
y chng ta s xem xt mt s v d p dng.
V d 6. Cho x, y, z l cc s thc dng tho mn iu kin x + y + z + 1 = 4xyz. Chng
minh rng
xy + yz + zx x + y + z.
tng u tin ta ngh n l x l iu kin bng php th z =

x + y +1
vo bt ng
4 yz 1

thc cn chng minh, vit li thnh

x + y +1
( x + y 1) x + y xy
4 xy 1

n y, ng li chng minh hnh thnh. V tri r rng 1 vi mi x, y cn v


phi s 1 nu x, y nm cng pha nhau i vi 1. Nhng iu ny, theo nhn xt n
gin trn ta lun c th chn c.
V d 7. Cho a, b, c l cc s thc khng m. Chng minh rng
(a2 + 2)(b2 + 2)(c2 + 2) 3(a + b + c)2
p dng bt ng thc CBS, ta c
(a + b + c)2 (a2 + 1 + 1)(1 + b2 + c2) = (a2 + 2)(b2 + c2 + 1). Nh vy ta ch cn cn
chng minh
(b2 + 2)(c2 + 2) 3(b2 + c2 + 1)
27 | Trn Nam Dng 6/2010

Vietnamese IMO Team Training Camp 2010



(b2 1)(c2 1) 0
iu ny lun c c nu ta chn b2, c2 cng pha nhau i vi 1.
Bi tp
5. Cho a, b, c > 0, x = a + 1/b, y = b + 1/c, z = c + 1/a. Chng minh rng
xy + yz + zx 2(x+y+z)
6. (USA MO 2001) Cho a, b, c l cc s thc khng m tho mn iu kin a2 + b2 + c2 + abc = 4. Chng
minh rng
0 ab + bc + ca abc 2.
7. Vi i = 1, 2, , 7 cc s ai, bi l cc s thc khng m tho mn iu kin
rng tn ti cc ch s i j sao cho |ai aj| + |bi bj| 1.

ai + bi 2. Chng minh

8. (VMO 1996) Cho x, y, z l cc s thc dng tho mn iu kin xy + yz + zx + xyz = 4. Chng minh
rng
x + y + z xy + yz + zx.
9. Tm s thc k nh nht sao cho xyz + 2 + k[(x-1)2 + (y-1)2 + (z-1)2] x + y + z vi mi x, y, z > 0.

Nguyn l Dirichlet trong t hp


T hp l mnh t mu m nht cho cc phng php v k thut chng minh. V
nguyn l Dirichlet khng phi l mt ngoi l. Trong t hp, mt c im c trng l
s bng n t hp ca cc trng hp, v vy, nguyn l Dirichlet cng vi cc nguyn l
khc nh nguyn l cc hn, nguyn l bt bin chnh l nhng cng c quan trng
chng ta nh hng trong bin cc trng hp.
Nguyn l Dirichlet thng c s dng trong cc bi ton th, t mu, cc bi ton
v thi u th thao ( th c hng), quen nhau ( th v hng).
V d 8. Trong mt gii bng chuyn c 8 i tham gia, thi u vng trn 1 lt. Chng
minh rng tm c 4 i A, B, C, D sao cho A thng B, C, D, B thng C, D v C thng
D.
Trong bng chuyn khng c ho, do 8 i thi u vng trn 1 lt th s c tt c 28
trn thng. Theo nguyn l Dirichlet, tn ti i bng A c t nht 4 trn thng. Xt 4 i
thua A. 4 i ny u vi nhau 6 trn, do tn ti mt i thng t nht 2 trn (trong s
cc trn u gia 4 i ny vi nhau). Gi s l B v C, D l 2 i thua B. Cui cng,
nu C thng D th A, B, C, D l 4 i cn tm, cn nu D thng C th 4 i cn tm l A,
B, D, C.
Bi ton Ramsey l mt trong nhng bi ton kinh in m nhng trng hp c s ca
n rt th v v ph hp vi mc ton s cp.
28 | Trn Nam Dng 6/2010

Vietnamese IMO Team Training Camp 2010


V d 9. Chng minh rng trong mt nhm 6 ngi bt k c 3 ngi i mt quen nhau
hoc 3 ngi i mt khng quen nhau.
Hai ngi bt k c th quen nhau hoc khng quan nhau. Xt mt ngi A bt k th c
5 ngi khc hoc quen A hoc khng quen A. Do , theo nguyn l Dirichlet, phi xy
ra mt trong hai trng hp:
1) A quen t nht 3 ngi
2) A khng quen t nht 3 ngi
Ta xt trng hp th nht: A quen vi t nht 3 ngi, chng hn B, C, D. Nu trong 3
ngi B, C, D c t nht mt cp quen nhau, chng hn B quen C th ta c A, B, C i
mt quen nhau. Trong trng hp ngc li, ta c B, C, D i mt khng quen nhau.
Trng hp th hai c xt hon ton tng t. Bi ton c chng minh.
V d 10. Trong mt nhm gm 2n+1 ngi vi mi n ngi tn ti mt ngi khc n
ngi ny quen vi tt c h. Chng minh rng trong nhm ngi ny c 1 ngi quen
vi tt c mi ngi.
Ta chng minh rng trong nhm ngi ny c n+1 ngi i mt quen nhau. R rng c
2 ngi quan nhau v nu nh c k ngi i mt quen nhau (trong k n) th tn ti
mt ngi khc trong s h quen vi k ngi ny. T suy ra tn ti n+1 ngi i mt
quen nhau A1, A2, , An+1.
Xt n ngi cn li. Theo iu kin,tn ti mt ngi Ai quen vi tt c n ngi ny.
Nhng khi Ai quen vi tt c mi ngi.
B quyt thnh cng ca nguyn l Dirichlet chnh l k thut xy chung v to th.
Trong nhiu bi ton, chung l g, th l g kh r rng, nhng trong nhiu bi ton, xy
chung v to th l c mt s tinh t. Ta phi bit chn cc thnh phn chnh v
hng n mc tiu.
V d 11. Cc s t 1 n 200 c chia thnh 50 tp hp. Chng minh rng trong mt
cc tp hp c ba s l di 3 cnh ca mt tam gic.
Thot nhn bi ton c v kh ri. Nhng nu rng vi 0 < a < b < c th iu kin
cn v a, b, c l di ba cnh ca mt tam gic l a + b > c th bi ton tr nn
n gin hn. R rng nu ch xt cc s t 100 n 200 th ba s bt k u l di 3
cnh ca 1 tam gic (a + b 100 + 101 = 201 > c). T ch cn xt 101 con th l cc
s t 100 n 200 ri p dng nguyn l Dirichlet cho 50 ci chung tp hp l xong.
y, r rng cc s t 1 n 99 ch c tc dng gy nhiu.
V d 12. Trn bn c quc t c 8 qun xe, i mt khng n nhau. Chng minh rng
trong cc khong cch i mt gia cc qun xe, c hai khong cch bng nhau. Khong
29 | Trn Nam Dng 6/2010

Vietnamese IMO Team Training Camp 2010


cch gia hai qun xe bng khong cch gia tm cc vung m qun cc qun xe
ng.
Trc ht ta m hnh ho bi ton. rng khong cch gia (p, q) v (m, n) bng
(p-m)2 + (q-n)2. Ta cn chng minh rng nu p1, p2, , p8 l mt hon v ca (1, 2, 3, ,
8) th tn ti cc tp ch s {m, n} {p, q} sao cho (m-n)2 + (pm-pn)2 = (p-q)2 + (pp pq)2.
8 qun xe to ra 28 khong cch. Nhng nu ta tm 2 khong cch bng nhau gia c 28
qun xe ny th ta s gp kh khn. Ta gii hn trong vic tm cc cp ch s dng {n,
n+1}. C 7 cp nh vy. Khi , ta ch cn tm n m sao cho (pn+1-pn)2 = (pm+1-pm)2. V
1 pi 8 nu (pn+1-pn)2 ch c th l 1 trong 7 gi tr 12, 22, , 72. V th ch c th xy
ra hai trng hp.
Trng hp 1. Tn ti n m sao cho (pn+1-pn)2 = (pm+1-pm)2. Khi cc cp qun xe ti
(n, pn), (n+1, pn+1) v (m, pm), (m+1, pm+1) l cc cp xe cn tm.
Trng hp 2. Cc s (pn+1-pn)2 i mt phn bit. Khi tn ti n sao cho
4.

(pn+1-pn)2 =

Lc ny, xoay hng thnh ct, ta li i n vic hoc tn ti n m sao cho (qn+1-qn)2 =
(qm+1-qm)2 hoc tn ti k sao cho (qk+1-qk)2 = 22. Trong trng hp th nht, bi ton c
gii quyt tng t nh trng hp 1 trn. Trong trng hp th
hai, cc qun xe ti (n, pn), (n+1, pn+1) v (qk+1,k+1), (qk, k) l cc cp xe cn tm.
Bi tp
10. Cc s 1, 2, 3, , 100 c th l thnh vin ca 12 cp s nhn no c khng?
11. Trong mt a gic li c cha khng t hn m2+1 im nguyn. Chng minh rng trong a gic li
ny tm c m+1 im nguyn cng nm trn mt ng thng.
12. Chng minh rng trong 9 ngi bt k, hoc c 3 ngi i mt quen nhau, hoc c 4 ngi i mt
khng quen nhau.
13. Chn ra 69 s nguyn dng t tp hp E = {1, 2, , 100}. Chng minh rng tn ti 4 s a < b < c
< d trong 4 s c chn sao cho a + b + c = d. Kt lun bi ton cn ng khng nu ta thay 69 bng 68?
14. Cc vung ca bng 100 x 100 c t bng 4 mu sao cho trn mi hng v trn mi ct c ng
25 c cng mt mu. Chng minh rng tn ti 2 dng v 2 ct sao cho bn nm giao ca chng
c t khc mu.
15. (Bulgarian MO 2005) Trong 9 ngi khng c 4 ngi no i mt quen nhau. Chng minh rng c
th chia 9 ngi ny thnh 4 nhm sao cho trong mi nhm khng c ngi no quen nhau.

30 | Trn Nam Dng 6/2010

Vietnamese IMO Team Training Camp 2010

Nguyn l Dirichlet trong hnh hc


Trong hnh hc, nguyn l Dirichlet thng c s dng trong cc bi ton lin quan
n di cnh, din tch, ln ca gc, cc bi ton trn li nguyn. y chng ti
ch gii hn trong vic gii thiu mt ng dng p ca nguyn l Dirichlet v chng
hnh trong hnh hc v mt s bi tp.
nh l Minkowsky l mt v d rt th v v ng dng ca hnh hc trong l thuyt s.
Chng ta bt u t mt kt qu rt n gin nhng hu ch
B 1. Trn mt phng cho hnh F c din tch ln hn 1. Khi tn ti hai im A, B
thuc F, sao cho vc t AB c ta nguyn.
Chng minh: Li nguyn ct hnh G thnh cc mu nh. Chng cc mu ny ln nhau,
do tng din tch ca cc mu ln hn 1, nn c t nht 2 mu c im chung (y chnh
l mt bin th ca nguyn l Dirichlet). Gi A, B l hai im nguyn thu ng vi im
chung ny th A, B l hai im cn tm.
B 2. (B Minkowsky) Trn mt phng cho hnh li F nhn gc ta lm tm i
xng v c din tch ln hn 4. Khi n cha mt im nguyn khc gc ta .
Chng minh: Xt php v t tm O, t s 1/2 , bin F thnh G. Do G c din tch ln
hn 1 nn theo b 1, tn ti hai im A, B thuc G sao cho vc-t AB c to
nguyn. Gi A l im i xng vi A qua O. Do hnh G i xng qua gc to nn
A thuc G. Do G li nn trung im M ca AB thuc G. Gi N l im i xng ca O
qua M th N thuc F v ON = AB, suy ra N l im nguyn khc O (pcm).
nh l 3. (nh l Minkowsky) Cho a, b, c l cc s nguyn, trong a > 0 v
= 1. Khi phng trnh ax2 + 2bxy + cy2 = 1 c nghim nguyn.

ac - b2

Bi tp
15. Vi gi tr no ca n tn ti n im M1, M2, , Mn sao cho tt c cc gc MiMjMk u khng t?
16. Cho 9 im nm trong hnh vung cnh 1. Chng minh rng tn ti mt tam gic c nh ti cc im
cho c din tch khng vt qu 1/8.

c thm bi Nguyn l chung v th

31 | Trn Nam Dng 6/2010

Vietnamese IMO Team Training Camp 2010


4. m bng hai cch
m bng hai cch l mt k thut thng dng to ra cc phng trnh, ng thc, cc
mi lin h gip chng ta gii quyt cc bi ton phng trnh, tnh ton hnh hc,
phng trnh hm, bt ng thc v c bit l cc bi ton t hp, trong c bi ton
m.
m bng hai cch v phng trnh hm
(Xem bi Gii phng trnh hm bng cch lp phng trnh)
m bng hai cch trong chng minh bt ng thc
Trong chng minh bt ng thc, m bng hai cch thng c s dng x l mt
s tng. Di y ta xem xt hai v d.
V d 1. Gi d l hiu gia s ln nht v s nh nht gia n s thc x1, x2, , xn
(n 2). Chng minh rng ta c bt ng thc
(n 1)d

1 i < j n

| xi x j |

n2d
4

Gii.
Khng mt tnh tng qut, gi s x1 x2 xn v t di = xi+1 - xi vi i = 1, 2, , n.
th di 0 v d = d1 + d2 + + dn-1. Vi i < j ta c |xi xj | = di + + dj-1
Mi dk s c mt trong tng di + + dj-1 nu i k v j-1 k, tc l trong k(n-k) tng.
T suy ra
n 1

1i < j n

| xi x j |= k (n k )d k
k =1

By gi ch n-1 k(n-k) n2/4 vi mi k = 1, 2, , n-1, ta suy ra bt ng thc cn


chng minh.
V d 2. Trong cc ca hnh vung 10 x 10 ta in cc s t 1 n 100 nh sau: hng
th nht in cc s t 1 n 10 t tri sang phi, hng th hai in cc s t 11 n 20
t tri sang phi An d nh ct hnh vung thnh cc hnh ch nht 1 x 2, ly tch ca
hai s trn mt hnh ch nht v cng tt c cc kt qu thu c vi nhau. An mun
cho tng thu c l nh nht c th. Hi An phi ct th no t c iu .
Gii.
Gi (ai, bi) i = 1, 2, , 50 l b s cc m An d nh ct. Ta c
50

50

i =1
50

i =1

ai bi =

Tng

(a
i =1

2
i

ai2 + bi2 (ai bi )


1 50
1 50
2
= (ai2 + bi2 ) (ai bi )
2
2 i =1
2 i =1
2

+ bi2 ) bng tng bnh phng cc s t 1 n 100, do khng i. Nh vy

32 | Trn Nam Dng 6/2010

Vietnamese IMO Team Training Camp 2010


50

ai bi t gi tr nh nht khi
i =1

50

(a b )
i =1

ln nht. Nhng ai bi = 1 nu hai ny nm

trn mt hng v ai bi = 10 nu hai ny nm trn mt ct. T suy ra An phi ct


thnh cc dc th tng thu c s nh nht.
Bi tp
1. Cc hm s f(x) v g(x) xc nh trn tp hp cc s nguyn, c tr tuyt i khng vt qu 1000.
Gi m l s cc cp (x, y) sao cho f(x) = g(y), n l s cc cp (x, y) sao cho f(x)=f(y), v k l s cc cp x,
y sao cho g(x)=g(y). Chng minh rng 2m n + k.
2. (IMO 2003) Cho n > 2 v cc s thc x1 x2 ... xn, chng minh rng

n
| xi x j

i , j =1

2
| (n 2 1) ( xi x j ) 2 .
3
i , j =1

Chng minh rng du bng xy ra khi v ch khi dy s l cp s cng.

C n2 trong cc bi ton t hp

Trong t hp C n2 l s cc cp to thnh t n phn t, l s cnh ca th y bc n.


Trong nhiu bi ton, s dng ngha t hp ny cng vi cch m bng hai cch gip
chng ta tm ra cha kho cho li gii.
V d 3. (Bulgarian MO 2006) Mt quc gia c 16 thnh ph v c 36 tuyn bay ni gia
chng. Chng minh rng ta c th t chc mt chuyn bay vng quanh gia 4 thnh ph.
Gii. Bi ton c th dch sang ngn ng th nh sau: Mt th G n c 16 nh v
36 cnh th c mt chu trnh di 4. Ta chng minh bng phn chng. Gi s trong
th khng c chu trnh di 4.
Vi mi nh x thuc G, gi d(x) l bc ca nh x. Khi s cp cc nh m c hai
cng k vi x bng C d2( x ) . Ch rng mi mt cp c tnh nhiu nht mt nh x,
v nu khng ta s c mt chu trnh di 4.
S dng ng thc

d ( x) = 72 v bt ng thc Cauchy-Schwarz, ta c
xG

C = 120 C
2
16

xG

2
d ( x)

d 2 ( x)
d ( x) 1
1
72 2

d ( x) d ( x) =
36 = 126 ,
2
2
32 xG
2 xG
32
xG
xG

mu thun.

V d 4. (IMO ) Trong mt k thi c a th sinh v b gim kho, trong b 3 l mt s


nguyn l. Mi mt gim kho s nh gi th sinh l u hay rt. Gi s k l mt s
33 | Trn Nam Dng 6/2010

Vietnamese IMO Team Training Camp 2010


nguyn sao cho vi hai gim kho bt k th nh gi ca h trng nhiu nht k th
sinh. Chng minh rng k/a (b-1)/2b.
Gii.
Ta m s N cc b ba (GK, GK, TS) trong hai gim kho l khc nhau v nh gi
th sinh ging nhau. C b(b-1)/2 cp gim kho v mi mt cp gim kho nh vy nh
gi trng nhau ti khng qu k th sinh, do N kb(b-1)/2.
By gi xt mt th sinh X v m cc cp gim kho nh gi X ging nhau. Gi s c x
gim kho cho X u, th s c x(x-1)/2 cp cng cho X u v (b-x)(b-x-1)/2 cp cng
cho X rt, nh vy c (x(x-1) + (b-x)(b-x-1))/2 cp nh gi X ging nhau. Nhng (x(x1) + (b-x)(b-x-1))/2 = (2x2 - 2bx + b2 - b)/2 = (x - b/2)2 + b2/4 - b/2 b2/4 - b/2 = (b 1)2/4 - 1/4. V (b - 1)2/4 l s nguyn (do b l), nn s cp nh gi X ging nhau s t
nht l (b - 1)2/4. T suy ra N a(b - 1)2/4. Kt hp hai bt ng thc li ta c
k/a (b - 1)/2b.
Bi tp
3. Trong Duma quc gia c 1600 i biu, lp thnh 16000 y ban, mi y ban c 80 i biu. Chng
minh rng c t nht hai y ban c khng di 4 thnh vin chung.
4. Sau khi khai trng c ng 10 ngy, mt nhn vin th vin cho bit :
1) Mi ngy c ng 8 ngi n c sch ;
2) Khng c ngi no n th vin 1 ngy qu 1 ln ;
3) Trong hai ngy bt k ca 10 ngy th c t nht l 15 ngi khc nhau cng n th vin.
Cn c ng thi c 3 iu kin m nhn vin th vin cung cp hy cho bit s ngi ti thiu
n th vin trong 10 ngy ni trn l bao nhiu ?
5. 2n k th thi u vng trn hai lt, mi lt hai k th bt k thi u vi nhau ng mt trn (thng
c 1 im, thua 0 im v ha 1/2 im). Ngi ta nhn thy rng lt hai, tng im ca mi k th
thay i khng nh hn n. Chng minh rng cc thay i ny ng bng n.
6. (Bulgarian MO 2003) Trong mt nhm n ngi c 3 ngi i mt quen nhau v mi mt ngi ny
quen nhiu hn 1 na s ngi trong nhm. Tm s t nht c th s b ba ngi i mt quen nhau.

m bng hai cch v mt s nh l trong l thuyt ti u t hp


Cho F l h cc tp con ca X. Vi x thuc x, ta gi d(x) l s phn t ca F cha x.
nh l 1. Cho F l h cc tp con ca tp hp X. Khi
Chng minh. Xt ma trn k M = (mx,A) ca F. Ngha l M l ma trn 0-1 vi |X| dng
nh s bi cc im x X v |F| ct nh s bi tp A F sao cho mx,A = 1 khi v ch
34 | Trn Nam Dng 6/2010

Vietnamese IMO Team Training Camp 2010


khi x A. rng d(x) bng s s 1 trn dng th x cn |A| l s s 1 trn ct th A.
Nh vy c v tri v v phi u biu din s s 1 ca M.
Nu ta xt th G = (V, E) trn tp nh V nh mt h cc tp con 2 phn t ca V th ta
c nh l Euler.
nh l 2. (Euler, 1736) Trong mi th, tng bc cc nh ca n bng hai ln s cnh
ca n v nh th, lun l mt s chn.
nh l sau c th c chng minh bng cch tng t
vi mi Y X.

nh l 3.

(Hai tng ng thc u ng vi s s 1 trn cc hng Y. Cc tng ng thc th hai


m s ln xut hin ca x trong cc tp c dng A B).
Trng hp c bit khi F = E l tp con 2 phn t, ta c
nh l 4. Vi th G = (V, E), ta c

(Xem thm bi Cc bi ton ti u v h cc tp hp)


5. Nguyn l cc hn
Mt tp hp hu hn cc s thc lun c phn t ln nht v phn t nh nht. Mt tp
con bt k ca N (hoc Nk) lun c phn t nh nht. Nguyn l n gin ny trong
nhiu trng hp rt c ch cho vic chng minh. Hy xt trng hp bin! l khu
quyt ca nguyn l ny.
Mt s v d m u
Ta xem xt mt s v d s dng nguyn l cc hn
V d 1. C 3 trng hc, mi trng c n hc sinh. Mi mt hc sinh quen vi t nht
n+1 hc sinh t hai trng khc. Chng minh rng ngi ta c th chn ra t mi trng
mt bn sao cho ba hc sinh c chn i mt quen nhau.
Gii.
Gi A l hc sinh c nhiu bn nht mt trng khc. Gi s ny l k. Gi s A
trng 1 v nhng bn quen A l B1, B2, ..., Bk trng 2. Ta c k

n +1
. Cng theo gi
2

thit, c t nht 1 hc sinh C trng 3 quen vi A. Gi s C khng quen vi Bi vi mi


i=1, 2, ..., k th C quen vi nhiu nht n - k hc sinh ca trng 2. Suy ra C quen vi t
nht n+1 - (n-k) = k+1 hc sinh trng 1, iu ny mu thun vi cch chn A. Vy C
phi quen vi mt Bi no . Khi A, Bi v C chnh l 3 hc sinh cn tm.
35 | Trn Nam Dng 6/2010

Vietnamese IMO Team Training Camp 2010

V d 2. Chng minh rng khng tn ti s n l, n > 1 sao cho 15n + 1 chia ht cho n
Gii. Gi s tn ti mt s nguyn l n > 1 sao cho 15n + 1 chia ht cho n. Gi p l c s
nguyn t nh nht ca n, khi p l. Gi s k l s nguyn dng nh nht sao cho 15k
1 chia ht cho p.
V 152n 1 = (15n-1)(15n+1) chia ht cho p. Mt khc, theo nh l nh Fermat th 15p-1
1 chia ht cho p. Theo nh ngha ca p, suy ra k l c s ca cc s p-1 v 2n. Suy ra k |
(p-1, 2n). Do p l c s nguyn t nh nht ca n nn (n, p-1) = 1. Suy ra (p-1, 2n) = 2.
Vy k | 2. T k = 1 hoc k = 2. C hai trng hp ny u dn ti p = 7. Nhng iu
ny mu thun v 15n + 1 lun ng d 2 mod 7
Bi tp
1. Cho n im xanh v n im trn mt phng, trong khng c 3 im no thng hng. Chng minh
rng ta c th ni 2n im ny bng n on thng c u mt khc mu sao cho chng i mt khng
giao nhau.
2. Trn ng thng c 2n+1 on thng. Mi mt on thng giao vi t nht n on thng khc. Chng
minh rng tn ti mt on thng giao vi tt c cc on thng cn li.
3. Trong mt phng cho n > 1 im. Hai ngi chi ln lt ni mt cp im cha c ni bng mt
vc-t vi mt trong hai chiu. Nu sau nc i ca ngi no tng cc vc t v bng 0 th ngi
th hai thng; nu cho n khi khng cn v c vc t no na m tng vn cha c lc no bng 0 th
ngi th nht thng. Hi ai l ngi thng cuc nu chi ng?

Nguyn l cc hn v bt ng thc
Nguyn l cc hn thng c p dng mt cch hiu qu trong cc bt ng thc c
tnh t hp, dng chng minh tn ti k s t n s tha mn mt iu kin ny .
V d 1. (Moscow MO 1984) Trn vng trn ngi ta xp t nht 4 s thc khng m c
tng bng 1. Chng minh rng tng tt c cc tch cc cp s k nhau khng ln hn .
Gii.
Ta cn chng minh rng vi mi n 4 s thc khng m 1, ..., n, c tng bng 1, ta c
bt ng thc
a1a2 + a2a3 + ... + an - 1an + ana1 1/4.
Vi n chn n (n = 2m) iu ny c th chng minh d dng: t a1 + a3 + ... + a2m - 1 = a;
khi , r rng,
a1a2 + a2a3 + ... + an - 1an + ana1 (a1 + a3 + ... + a2m1) (a2 + a4 + ... + a2m) = a(1 a)
1/4.

36 | Trn Nam Dng 6/2010

Vietnamese IMO Team Training Camp 2010


Gi s n l v ak l s nh nht trong cc s cho. ( thun tin, ta gi s 1 < k < n
1 iu ny khng lm mt tnh tng qut khi n 4.) t bi = i, vi i = 1,..., k 1, bk
= ak + ak + 1 v bi = ai + 1 vi i = k + 1,..., n 1. p dng bt ng thc ca chng ta cho
cc s b1,..., bn - 1, ta c:
a1a2 + ... + ak - 2ak - 1 + (ak - 1 + ak + 2) bk + ak + 2ak + 3 + ... + an - 1an + ana1 1/4.
Cui cng, ta s dng bt ng thc
ak - 1ak + akak + 1 + ak + 1ak + 2 ak - 1ak + ak - 1ak + 1 + ak + 1ak + 2 (ak - 1 + ak + 2) bk.
suy ra iu phi chng minh.
nh gi trn y l tt nht; du bng xy ra khi 2 trong n s bng 1/2, cn cc s cn
li bng 0.
V d 2. Cho n 4 v cc s thc phn bit a1, a2, , an tho mn iu kin
n

a
i =1

= 0, ai2 = 1.
i =1

Chng minh rng tn ti 4 s a, b, c, d thuc {a1, a2, , an} sao cho


n

a + b + c + nabc ai3 a + b + d + nabd .


i =1

V d 3. Tng bnh phng ca mt 100 s thc dng ln hn 10000. Tng ca chng


nh hn 300. Chng minh rng tn ti 3 s trong chng c tng ln hn 100.
Gii. Gi s 100 s l C1 C2 ... C100 > 0. Nu nh C1 100, th C1 + C2 + C3 >
100. Do ta c th gi s rng C1 < 100. Khi 100 - C1 > 0, 100 - C2 > 0, C1 - C2 0
C1 - C3 0, v vy
100(C1 + C2 + C3) 100(C1 + C2 + C3) - (100 - C1)(C1 - C3) - (100 - C2)(C2 - C3) =
= C12 + C22 + C3(300 - C1 - C2) >
> C12 + C22 + C3(C3 + C4 + ... + C100)
C12 + C22 + C32 + ... + C1002 > 10000.
Suy ra, C1 + C2 + C3 > 100.
Bi tp
1. Trong mi ca bng 2 x n ta vit cc s thc dng sao cho tng cc s ca mi ct bng 1. Chng
minh rng ta c th xo i mi ct mt s sao cho mi hng, tng ca cc s cn li khng vt qu

n +1
.
4

2. 40 tn trm chia 4000 euro. Mt nhm gm 5 tn trm c gi l ngho nu tng s tin m chng
c chia khng qu 500 euro. Hi s nh nht cc nhm trm ngho trn tng s tt c cc nhm 5 tn
trm bng bao nhiu?

Nguyn l cc hn v phng trnh Diophant

37 | Trn Nam Dng 6/2010

Vietnamese IMO Team Training Camp 2010


Nguyn l cc hn ng dng trong phng trnh Diophant c nhc ti trong bi
phng php chng minh phn chng. Trong phn ny, ta trnh by chi tit ba v d p
dng nguyn l cc hn trong phng trnh Fermat, phng trnh Pell v phng trnh
dng Markov.
V d 1. Chng minh rng phng trnh x4 + y4 = z2 (1) khng c nghim nguyn dng.
Gi s ngc li, phng trnh (1) c nghim nguyn dng, v (x, y, z) l nghim ca
(1) vi z nh nht.
(1) D thy x2,y2,z i mt nguyn t cng nhau
(2) T nghim ca phng trnh Pythagore, ta c tn ti p, q sao cho
x2 = 2pq
y2 = p2 - q2
z = p2 + q2
(3) T y, ta li c mt b ba Pythagore khc, v y2 + q2 = p2.
(4) Nh vy, tn ti a,b sao cho
q = 2ab
y = a2 - b2
p = a2 + b2
a,b nguyn t cng nhau
(5) Kt hp cc phng trnh ny, ta c:
x2 = 2pq = 2(a2 + b2)(2ab) = 4(ab)(a2 + b2)
(6) V ab v a2 + b2 nguyn t cng nhau, ta suy ra chng l cc s chnh phng.
(7) Nh vy a2 + b2 = P2 v a = u2, b = v2. Suy ra P2 = u4 + v4.
(8) Nhng by gi ta thu c iu mu thun vi tnh nh nht ca z v:
P2 = a2 + b2 = p < p2 + q2 = z < z2.
(9) Nh vy iu gi s ban u l sai, suy ra iu phi chng minh.
V d 2. Tm tt c cc cp a thc P(x), Q(x) tha mn phng trnh
P2(x) = (x2-1)Q2(x) + 1 (1)
Gii. Khng mt tnh tng qut, ta ch cn tm nghim trong tp cc a thc c h s khi
u dng.
Nu ( x + x 2 1) n = Pn ( x) + x 2 1Qn ( x) (2) th ( x x 2 1) n = Pn ( x) x 2 1Qn ( x) (3)
38 | Trn Nam Dng 6/2010

Vietnamese IMO Team Training Camp 2010


Nhn (2) v (3) v theo v, ta c
1 = ( x + x 2 1) n ( x x 2 1) n = ( Pn ( x) + x 2 1Qn ( x))( Pn ( x) x 2 1Qn ( x))
= Pn2 ( x) ( x 2 1)Qn2 ( x)

Suy ra cp a thc Pn(x), Qn(x) xc nh bi (2) (v (3)!) l nghim ca (1). Ta chng


minh y l tt c cc nghim ca (1). Tht vy, gi s ngc li, tn ti cp a thc
P(x), Q(x) khng c dng Pn(x), Qn(x) tha mn (1). Ta xt cp a thc (P, Q) nh vy
vi degQ nh nht.
t ( P ( x) + x 2 1Q( x))( x x 2 1) = P * ( x) + x 2 1Q * ( x) (4)
Th r rng
( P ( x) x 2 1Q( x))( x + x 2 1) = P * ( x) x 2 1Q * ( x)

Suy ra (P*, Q*) cng l nghim ca (1).


Khai trin (4), ta thu c P*(x) = xP(x) (x2-1)Q(x), Q*(x) = xQ(x) P(x). Ch l t
(1) ta suy ra (P(x) xQ(x))(P(x)+xQ(x)) = - Q2(x) + 1. V P(x) v Q(x) u c h s khi
u > 0 v degP = degQ + 1 nn ta c deg(P(x)+xQ(x)) = degQ + 1. T y, do deg(Q2(x) + 1) 2deg(Q) nn ta suy ra deg(Q*(x)) deg(Q) 1 < deg Q.
Nh vy, theo cch chn cp (P, Q) th tn ti n sao cho (P*, Q*) = (Pn, Qn).
Nhng khi t (4) suy ra
P( x) + x 2 1Q( x) = ( P * ( x) + x 2 1Q * ( x))( x + x 2 1)
= ( x + x 2 1) n ( x + x 2 1) = ( x + x 2 1) n +1

Suy ra (P, Q) = (Pn+1,Qn+1), mu thun.


Vy iu gi s l sai v ta c iu phi chng minh.
V d 3. Tm tt c cc gi tr k sao cho phng trnh (x+y+z)2 = kxyz c nghim nguyn
dng.
V d 4. (CRUX, Problem 1420) Nu a, b, c l cc s nguyn dng sao cho
0 < a2 + b2 abc c
Chng minh rng a2 + b2 abc l s chnh phng.
Gii. Gi s ngc li rng tn ti cc s nguyn dng a, b, c sao cho 0 < a2 + b2 abc
c v k = a2 + b2 abc (1) khng phi l s chnh phng.
By gi ta c nh k v c v xt tp hp tt c cc cp s nguyn dng (a, b) tha mn
phng trnh (1), tc l ta xt
S(c, k) = {(a, b) (N*)2: a2 + b2 abc = k}
Gi s (a, b) l cp s thuc S(c, k) c a + b nh nht. Khng mt tnh tng qut c th
gi s a b. Ta xt phng trnh
39 | Trn Nam Dng 6/2010

Vietnamese IMO Team Training Camp 2010


x2 bcx + b2 k = 0
Ta bit rng x = a l mt nghim ca phng trnh. Gi a1 l nghim cn li ca phng
trnh ny th a1 = bc a = (b2 k)/a.
Ta c th chng minh c rng (bn c t chng minh!) a1 nguyn dng. Suy ra
(a1, b) cng thuc S(c, k).
Tip theo ta c a1 = (b2-k)/a < a2/a = a, suy ra a1 + b < a + b. iu ny mu thun vi cch
chn (a, b).
Bi tp
1. (IMO 88) Nu a, b, q = (a2+b2)/(ab+1) l cc s nguyn dng th q l s chnh phng.
2. (PTNK 03). Tm tt c cc s nguyn dng k sao cho phng trnh x2 - (k2-4)y2 = - 24 c nghim
nguyn dng.
3. (Mathlinks) Cho A l tp hp hu hn cc s nguyn dng. Chng minh rng tn ti tp hp hu hn
cc s nguyn dng B sao cho A B v xB x = xB x2.
4. (AMM 1995) Cho x, y l cc s nguyn dng sao cho xy + x v xy + y l cc s chnh phng.
Chng minh rng c ng mt trong hai s x, y l s chnh phng.
5. (IMO 2007) Cho a, b l cc s nguyn dng sao cho 4ab 1 chia ht (4a2-1)2. Chng minh rng a = b.

6. Sp th t
Sp th t l mt cch gim s trng hp cn xt v to ra cc iu kin b sung
gip chng ta c th lm vic d dng hn.
V d 1. Cho x1, x2, , xn l n s thuc on [0, 2]. Chng minh rng
n

| x
i =1 j =1

x j | n2.

V d 2. (Bt ng thc Schur m rng) Cho x, y, z l cc s thc khng m, r l mt s


thc dng bt k. Chng minh rng ta c bt ng thc
ar(a-b)(a-c) + br(b-a)(b-c) + cr(c-a)(c-b) 0
V d 3. Cho a, b, c l cc s thc i mt khc nhau. Chng minh rng
1
1
1 9

a 2 + b 2 + c 2
+
+
2
2
2
a

b
b

c
c

a
(
)
(
)
(
)

Bi tp
1. Cho x, y, z thuc [0, 1]. Chng minh rng (x+y+z)((x-y)2+(y-z)2+(z-x)2) 4.

40 | Trn Nam Dng 6/2010

Vietnamese IMO Team Training Camp 2010

2. (IMO 2006) Tm s thc M nh nht sao cho bt ng thc


|ab(a2 - b2) + bc(b2 - c2) + ca(c2 - a2)| < M(a2 + b2 + c2)2
ng vi mi s thc a, b, c.
3. (Vietnam MO 2008) Cho x, y, z l cc s thc khng m khc nhau. Chng minh rng

1
1
1
( xy + yz + zx)
+
+
4.
2
2
( y z)
( z x) 2
( x y)

Ti liu tham kho


1. on Qunh, Don Minh Cng, Trn Nam Dng, ng Hng Thng, Ti liu gio
khoa chuyn ton, i s 10, Nh xut bn Gio dc 2009.
2. on Qunh, Don Minh Cng, Trn Nam Dng, ng Hng Thng, Ti liu gio
khoa chuyn ton, Bi tp i s 10, Nh xut bn Gio dc 2009.
3. Nguyn Vn Mu, Trn Nam Dng, V nh Ha, ng Huy Run, Chuyn chn
lc T hp v Ton ri rc, Nh xut bn Gio dc 2008.
4. Arthur Engel, Problem Solving Strategies, Springer 1998.
5. N.Agakhanov, Cc bi thi Olimpic Ton ton Nga 1996-2006, Nh xut bn MCCME
2007.
6. A. Kovaldzi v A. Kanel-Belov, Gii bi ton khng mu mc nh th no, Nh xut
bn MCCME 2006.
7. Stasys Jukna, Extremal Combinatorics, Springer 2001.

41 | Trn Nam Dng 6/2010

Vietnamese IMO Team Training Camp 2010

Nguyn l chung v th
Nguyn l chung v th (hay cn c gi l nguyn l Dirichlet) khng nh mt s
kin hin nhin rng n+1 con th khng th c xp vo n chung sao cho mi con
th u ring mt chung. Mt cch tng qut hn, nguyn l chung v th khng
nh rng:
Nu mt tp hp gm nhiu hn kn i tng c chia thnh n nhm, th c mt nhm
no c nhiu hn k i tng.
Chn l ny rt d kim tra: nu nhm no cng c nhiu nht k i tng th tng cng
ch c nhiu nht kn i tng c chia ra.
y l mt trong nhng nguyn l khng xy dng (non-constructive) lu i nht: n
ch ni n s tn ti ca mt chung trong c nhiu hn k vt m khng ni g n
cch tm ra chung ny. Ngy nay chng ta c nhng tng qut ha rt mnh ca
nguyn l ny (cc nh l kiu Ramsey, phng php xc sut).
Mc d nguyn l chung v th c pht biu rt n gin, n c hng lot cc ng
dng khng tm thng. Ci kh ca vic ng dng nguyn l ny l xc nh c xem
th l g v chung l g. Chng ta s minh ha iu ny bng mt s v d.
1. Mt s v d m u
khi ng, chng ta s bt u bng nhng ng dng n gin nht. Bc ca mt nh
trong th G l s d(x) cc cnh ca G k vi x.
Mnh 1. Trong mi th tn ti hai nh c cng bc.
Chng minh. Gi s ta c th G c n nh. Ta to ra n ci chung c nh s t 0
n n-1 v xp nh x vo chung th k khi v ch khi d(x) = k. Nu nh trong mt
chung no c nhiu hn 1 nh th ta c pcm. V th ta c th gi s rng khng c
chung no cha hn 1 nh. C tt c n nh c chia vo n ci chung, nhng vy mi
mt chung c ng 1 nh. Gi x v y l cc nh nm trong cc chung nh s 0 v n1 tng ng. nh x c bc 0 v vy n khng c ni vi cc nh khc, trong c y.
Nhng y c bc n-1 nn n li c ni vi tt c cc nh, trong c x, mu thun.
Nu G l mt th hu hn, ch s c lp (independent number) (G) l s ln nht
cc nh i mt khng k nhau ca G. Sc s (chromatic number) (G) ca G l s nh
nht cc mu cn dng t cc mu ca G sao cho khng c hai nh k nhau c t
cng mu.
42 | Trn Nam Dng 6/2010

Vietnamese IMO Team Training Camp 2010

Mnh 2. Trong mi th G vi n nh ta c n (G).(G).


Chng minh. Ta chia cc nh ca G thnh (G) nhm (cc tp hp cc nh c cng
mu). Theo nguyn l chung v th, mt trong cc nhm c cha t nht n/(G) nh,
v cc nh ny i mt khng k nhau. Nh vy (G) n/(G) v chnh l iu cn
chng minh.
Mt th l lin thng nu gia hai nh bt k ca n c mt ng i.
Mnh 3. Cho G l mt th n nh. Nu mi nh ca G c bc t nht l
th G lin thng.

(n-1)/2

Chng minh. Ta xt hai nh x, y bt k. Nu hai nh ny khng k nhau th c t nht n1 nh ni chng vi cc nh cn li, v c x v y u c bc t nht l (n-1)/2. V ch cn
n-2 nh khc, nguyn l chung v th suy ra rng phi c mt trong cc nh ni vi
c x v y. Ta chng minh c rng mi cp nh th hoc k nhau, hoc c nh k
chung, v nh vy G lin thng.
Ghi ch. Mt kt qu l tt nht nu nh kt lun khng cn ng khi ta lm yu i mt
iu kin. V d, trong kt qu trn: gi s n l chn v G l hp ca hai th y
vi n/2 nh th bc ca mi nh bng (n-2)/2 nhng th khng lin thng.
Bi tp 1. Gi s 5 im c chn trong hnh vung cnh 1. Chng minh rng tn ti t nht 1 cp im
cch nhau khng qu 1/2.
Bi tp 2. Cc vin ca 8 mu khc nhau c xp vo 6 ci hp. C 20 vin cho mi mu. Chng
minh rng tm c mt hp cha hai cp c cng mu khc nhau.
Bi tp 3. Chng minh rng mt tp hp bt k gm n+1 phn t c chn t {1, 2,,2n} u cha
mt cp phn t c tng bng 2n+1. Hy chng minh kt qu ny l tt nht.
Bi tp 4. Chng minh rng mt tp hp bt k gm n+1 s nguyn c chn t {1, 2,, 2n} cha hai
s m s ny chia ht cho s kia.

2. nh l Erdos-Szekeres
Cho A = (a1, a2,, an) l dy gm n s phn bit. Mt dy con k phn t ca A l dy B
gm k s hng phn t ca A xut hin theo ng th t m chng xut hin trong A. C
ngha l B = (ai1, ai2,, aik) vi i1 < i2 < < ik. Dy con B c gi l tng nu ai1 < ai2
<< aik, v gim nu ai1 > ai2 >> aik.
Ta quan tm n di ln nht ca dy con tng v gim ca A. Suy lun trc quan cho
thy phi c mt s cn i nht nh gia hai di ny. Nu nh dy con tng di nht
l ngn, chng hn c chiu di l s, th mi dy con ca A c di s+1 phi cha cp
43 | Trn Nam Dng 6/2010

Vietnamese IMO Team Training Camp 2010


phn t gim, nh vy c rt nhiu cp phn t gim. V th ta trng i rng dy con
gim di nht s ln. Mt trng hp cc bin xy ra khi s = 1. Khi c dy s A l
gim.
Lm sao ta c th s ha iu d cm rng di ca dy con tng di nht v dy con
gim di nht khng th cng nh ? Kt qu ni ting ca Erdos v Szekeres (1935) cho
chng ta cu tr li cho cu hi ny v y l mt trong nhng kt qu u tin ca ti u
t hp.
nh l 4 (Erdos-Szekeres 1935). Cho A = (a1, a2,, an) l dy gm n s thc phn bit.
Nu n rs + 1 th hoc A c dy con tng di s+1 hoc A c dy con gim di r+1
(hay c hai).
Chng minh. (ca Seidenberg 1959). Ta cho tng ng mi phn t ai ca A vi cp
im s (xi, yi) trong xi l s phn t ca dy con tng di nht kt thc ti ai v yi l
s phn t ca dy con gim di nht bt u t ai. Ch rng khng c hai phn t no
c cng im s, tc l (xi, yi) (xj, yj) vi mi i j. Tht vy, nu ta c ... ai ... aj ..., th
hoc ai < aj v dy con tng di nht kt thc ti ai c th ko di n aj (v do xi < xj),
hoc ai > aj v dy con gim di nht bt u t aj c th c bt u t ai (v nh th yi
> yj).
By gi ta to ra mt li gm n chung th.
n

1
1

Ta t mi phn t ai vo chung vi ta (xi, yi). Mi mt phn t ca A c th c


t vo mt chung v 1 xi, yi n vi mi i = 1, 2, ..., n. Hn na, khng c chung
no c cha nhiu hn mt phn t, v (xi, yi) (xj, yj) vi mi i j. V |A| = n rs +
1, ta c nhiu vt hn l s chung th c t m trong hnh v trn. Nh vy phi c
mt phn t ai nm ngoi min t m. Nhng iu ny c ngha l xi s+1 hoc yi r +
1 (hoc c hai), ng iu chng ta cn.
Tp hp cc s thc c sp ton phn. iu ny c ngha l vi hai s phn bit x, y
th hoc x < y hoc y < x. B di y, thuc v Dilworth, s tng qut ha nh l
Erdos-Szekeres cho cc tp hp m trong hai phn t c th khng so snh c.
44 | Trn Nam Dng 6/2010

Vietnamese IMO Team Training Camp 2010

Mt th t b phn (yu) trn tp hp P l quan h hai ngi < gia cc phn t ca P. Ta


ni hai phn t x v y l so snh c nu x < y hoc y < x (hoc c hai). Mt xch l
mt tp hp Y P sao cho hai phn t bt k ca Y l so snh c. Nu khng c hai
phn t khc nhau no ca Y l so snh c, th Y c gi l i xch.
B 5 (Dilworth 1950). Trong mi th t b phn trn tp hp P gm n sr + 1 phn
t, tn ti xch c kch thc s+1 hoc i xch c kch thc r+1.
Chng minh. Gi s rng khng c xc di s+1. Khi ta c th nh ngha hm s f:
P  {1,..., s} trong f(x) l s phn t ln nht ca mt xch c phn t ln nht x.
Theo nguyn l chung v th, s c r+1 phn t ca P c cng nh qua nh x f. Theo
nh ngha ca f, cc phn t ny khng so snh c; v nh vy chng to thnh mt
i xch c kch thc r+1.
Bi tp 5. T b Dilworh hy suy ra nh l Erdos-Szekeres.
Bi tp 6. Cho n2+1 im trong mt phng. Chng minh rng tn ti dy gm n+1 im
(x1,y1),(x2,y2),,(xn+1,yn+1) sao cho x1 x2 xn+1 v y1 y2 yn+1, hoc dy gm n+1 im sao
cho x1 x2 xn+1 v y1 y2 yn+1

3. nh l Mantel
Di y chng ta s tho lun v mt tnh cht ti u c trng ca th. Mt th G
gm 2n nh khng cha tam gic c th c bao nhiu cnh? Tam gic l tp hp {x, y,
z} gm ba nh m hai nh bt k u c ni vi nhau bi mt cnh. D nhin l G c
th cha n2 cnh m khng cha tam gic: ch cn ly th hai phe y gm hai tp
hp mi tp hp c n nh v tt c cc cnh ni gia hai tp hp. Thc t l n2 chnh l
s cnh ln nht c th: nu ta thm mt cnh v th th s xut hin tam gic.
Ta s a ra 4 chng minh cho kt qu p ny: chng minh th nht dng nguyn l
chung v th, chng minh th hai da trn phng php m bng hai cch, chng
minh th ba s dng bt ng thc AM-GM v chng minh th t s dng l lun dch
chuyn (ta s cp ti chng minh ny trong phn sau).
nh l 6 (Mantel 1907). Nu th G vi 2n nh c n2+1 cnh th G cha tam gic.
Chng minh th nht. Ta chng minh bng quy np theo n. Vi n = 1, th G khng th c
n2+1 cnh v v vy mnh ng. Gi s mnh ng n n, ta xt th G vi
2(n+1) nh v (n+1)2 + 1 cnh. Gi x v y l hai nh k nhau trong G, v H l th con
cm sinh trn 2n nh cn li. Nu H cha t nht n2+1 cnh th theo gi thit quy np, ta
c ngay pcm. Gi s rng H c nhiu nht n2 cnh, khi c t nht 2n+1 cnh ca G s
ni t x v y n cc nh ca H.
45 | Trn Nam Dng 6/2010

Vietnamese IMO Team Training Camp 2010


Theo nguyn l chung v th, gia 2n+1 cnh ny c t nht 1 cnh ni t x v mt cnh
ni t y n cng mt nh z thuc H. Nh vy G cha tam gic {x, y, z}.
Chng minh th hai. Cho G l th trn tp hp V gm 2n nh v c m n2+1 cnh.
Gi s rng G khng cha tam gic. Khi cc cnh k nhau khng c nh k chung, do
d(x) + d(y) 2n vi mi cnh {x, y} E. Cng theo tt c cc cnh ca G, ta c

d ( x)
xV

(d ( x) + d ( y)) 2mn.

{ x , y }E

Mt khc, s dng bt ng thc Cauchy-Schwarz v ng thc Euler

d ( x) = 2m , ta
xV

c
2

d ( x)
2
xV
= 2m .
2
d
(
x
)
=

|V |
n
xV

T hai bt ng thc ny suy ra m n2, mu thun vi gi thit.


Chng minh th ba. Gi s G = (V, E) l th trn tp V gm 2n nh v gi s G
khng cha tam gic. Gi s A V l tp hp c lp ln nht, tc l tp hp ln nht
cc nh sao cho khng c nh no k nhau trong G. V G khng cha tam gic tt c
cc nh k vi nh x V to thnh mt tp c lp v ta suy ra d(x) |A| vi mi x.
Tp hp B = V A giao vi mi cnh ca G. Tnh cc cnh ca G tng ng vi nh
cui ca chng trong B, ta c | E | xB d ( x) . By gi bt ng thc AM-GM cho ta
| A | + | B |
2
| E | xB d ( x ) | A | . | B |
=n .
2

4. nh l Turan
Mt k-clique l mt th vi k nh m hai nh bt k u c ni vi nhau bi mt
cnh. V d tam gic l 3-clique. nh l Mantel khng nh rng nu th vi n nh
khng cha 3-clique th n c nhiu nht n2/4 cnh. Cn nu k > 3 th sao?
Cu tr li c cho bi kt qu c bn ca Paul Turan, kt qu m u cho l thuyt
th ti u.
nh l 7 (Turan 1941). Nu th G = (V, E) trn n nh khng cha (k+1)-clique, k
2, th
2
1n
| E | 1 .
k 2

(1)

Cng ging nh nh l Mantel, nh l ny c nghin cu nhiu ln vi nhiu cc


chng minh khc nhau. y chng ta a ra chng minh nguyn thy ca Turan. Php
46 | Trn Nam Dng 6/2010

Vietnamese IMO Team Training Camp 2010


chng minh da trn l lun dch chuyn trng lng s c cp trong phn bi
tp. Ngoi ra s cn mt cch chng minh s dng mt tng hon ton khc l lun
xc sut.
Chng minh. Ta s dng php quy np theo n. Khi n = 2, bt ng thc (1) l hin nhin
ng. Trng hp k=2 chnh l nh l Mantel. By gi gi s bt ng thc ng cho
mi th trn nhiu nht n-1 nh, v G = (V, E) l th trn n nh khng c (k+1)clique v c s cnh ln nht. th ny d nhin l phi cha k-clique, bi nu khng ta
c th thm cnh. Gi s A l k-clique v B = V A.
k

V mi cp nh ca A c ni bi mt cnh, A cha e A = cnh. Gi eB l s cnh


2
ni cc nh ca B v eA,B l s cnh ni gia cc cnh ca A v B. Theo gi thit quy
np, ta c
2
1 (n k )
.
e B 1
2
k

V G khng c k+1 clique nn mi x B k vi nhiu nht k-1 nh thuc A, v ta thu


c
eA,B (k-1)(n-k).
Cng cc bt ng thc ny li v s dng ng thc
2
2
k n
1n

=
1
k 2 2 2

ta suy ra rng
k k n k
| E | e A + e B + e A, B +
+ (k 1)(n k )
2 2 2
2

2
k n k
1n
= 1
= 1 .
k
k 2
2
2

Bi tp 7. Gi s rng n l bi s ca k. Hy xy dng mt th khng cha (k+1)-clique, trong s


cc cnh t c cn trn (1) trong nh l 7.
Bi tp 8. Nhc li ch s c lp (G) ca th G l s ln nht cc nh i mt khng k nhau ca G.
Hy chng minh i ngu ca nh l Turan: Nu G l th vi n nh v nk/2 cnh, k 1, th (G)
n/(k+1).

5. nh l Dirichlet
y ta trnh by mt ng dng ca nguyn l chung v th m Dirichlet s dng,
v chnh v ng dng ny m nguyn l ny c gn vi tn ng. N lin quan n vn
tn ti xp x hu t tt cho cc s v t. Kt qu ny thuc v l thuyt s nhng l
lun l t hp.
47 | Trn Nam Dng 6/2010

Vietnamese IMO Team Training Camp 2010

nh l 8 (Dirichlet 1879). Nu x l mt s thc. Vi mi s nguyn dng n,tn ti s


hu t p/q sao cho 1 q n v
x

1
1
p
<
2.
q nq q

Chng minh. Cho chng minh ny, ta gi {x} l phn l ca s thc x, tc l {x} = x
[x].
Nu x l s hu t th khng c g chng minh. V th, ta gi s rng x l v t v xt
n+1 s {ax}, a = 1, 2, , n+1. Ta t n+1 s ny vo n chung
1 1 2 n 1
,1.
0, , , ,...,
n n n n

(Khng c s no trong cc s ni trn trng vi u mt cc on, v x l v t). Theo


nguyn l chung v th, c mt on no cha nhiu hn mt s, v d l {ax} v
{bx}vi a > b, v do cch nhau khng qu 1/n. t q = a b, ta thy rng tn ti s
nguyn p sao cho |qx p| < 1/n, t suy ra kt qu cn chng minh bng cch chia cho
q. Hn na, q l hiu ca hai s nguyn thuc 1, 2, , n+1, do q n.
6. th c t c sc
Ta t mu cc cnh ca th y Kn trn n nh. Ta ni rng th c t c sc
(swell-colored) nu mi tam gic cha 1 hoc 3 mu, nhng khng cha 2 mu v th
c nhiu hn mt mu. C ngha l, ta cn s dng t nht 2 mu v vi mi tam gic,
hoc l tt c cc cnh ca n c cng mu hoc c mu khc nhau.
Ta c th chng minh c rng (hy chng minh!) Kn khng th c t c sc vi
ng hai mu. Cng c th thy rng K3 v K4 l nhng th Kn duy nht c th t c
sc vi 3 mu; cc th Kn khc cn nhiu mu hn v chng c bc lin thng cao hn.
S dng nguyn l chung v th, ta c th chng minh c chn di sau.
nh l 9 (Ward-Szabo 1994). th y trn n nh khng th c t c sc vi t
hn n + 1 mu.
Chng minh. Gi s Kn c t c sc vi r mu khc nhau. Gi N(x, c) l s cnh k
vi nh x c t mu c. C nh x0 v c0 sao cho N(x0, c0) ln nht, v k hiu gi tr
ln nht ny l N.
n-1 cnh k vi x0 c th c chia thnh r nhm mu, mi nhm c N hoc t hn
phn t. Theo nguyn l chung v th
N.r n 1.
48 | Trn Nam Dng 6/2010

Vietnamese IMO Team Training Camp 2010


Gi x1, x2, , xN l cc nh k vi x0 bi N cnh c mu c0. Gi G l th con (y )
ca Kn cm sinh t tp hp cc nh {x0, x1,..., xN}. Tnh c sc ca Kn c cm sinh
cho G v nh vy mi cnh ca G c cng mu c0. V Kn c t nht hai mu nn tn ti
nh y thuc Kn khng nm trong G v sao cho t nht mt cnh ni y vi G c mu khc
c0 .
Khng nh 10. N+1 cnh ni y ti G c t mu khc nhau v khc vi c0.
T khng nh ny ta suy ra r N + 2, t , cng vi bt ng thc N.r n 1 suy ra
r(r-2) n 1, v t r n + 1 l iu cn chng minh. Nh vy ta ch cn cn chng
minh khng nh trn.
Nu mt cnh ni y ti G, v d {y, x1}, c mu c0 th theo tnh c sc, cnh {y,x0} phi
c t mu c0, mu thun vi cch chn y (nhc li l x1, x2,,xN l tt c cc nh k
vi x0 c ni bng cnh mu c0). Tip theo, nu hai cnh no ni y vi G, chng
hn {y, x1} v {y, x2} c cng mu th theo tnh c sc ca Kn ta c cnh {x1, x2} cng
c t bng mu . Nhng {x1, x2} thuc G v c mu c0 v nh vy {y, x1} phi c
mu c0 l iu m ta chng minh trn l khng th. iu ny kt thc php chng
minh khng nh v cng l kt thc chng minh nh l.
Tnh ti u ca cn di cho bi nh l 9 c th c chng t s dng mt cu hnh
gi l mt phng afine. Ta hiu rng mt phng afine AG(2,q) bc q cha ng q2 im
v ng q+1 lp (c gi l bt ch) cc ng thng song song, mi lp cha q
ng thng (hai ng thng l song song nu chng khng c im chung). Hn na,
hai im bt k nm trn ng mt ng thng.
Khi c 1 mt phng nh vy, ta c th xy dng mt php t c sc cho K q vi q+1
2

mu nh sau. Ta ng nht cc nh ca K q vi cc im ca AG(2,q) v cho tng ng


2

mt mu duy nht cho mi mt trong s q+1 bt ch cc ng thng song song. xc


nh cch t c sc, ta xt hai im khc nhau x v y ca K q . Hai im ny thuc duy
2

nht mt ng thng v ng thng ny, n lt mnh thuc duy nht mt bt ch. Ta


t mu cnh {x,y} bng mu ca bt ch ny. V hai im bt k nm trn mt ng
thng duy nht v hai ng thng song song khng c im chung, mi cnh ca mt
tam gic s c t bng cc mu khc v nh vy, php t l c sc nh mong mun.
Thc t, Ward v Szabo (1994) chng minh rng iu ngc li cng ng: nu th
K q (q 2) c th t c sc bng q+1 mu th php t ny c th c dng xy dng
2

mt phng afine bc q.
[Trch t cun Extremal Combinatorics ca Stasys Jukna]
49 | Trn Nam Dng 6/2010

Vietnamese IMO Team Training Camp 2010

Gii phng trnh hm bng cch lp phng trnh


Trn Nam Dng
Trng H KHTN Tp.HCM
Cuc sng l chui nhng phng trnh
m ta kim tm li gii
Gii bi ton bng cch lp phng trnh v h phng trnh l mt phng php thng
dng trong cc bi ton i s. tng l tm mt n s no , ta a vo cc n s
ph, s dng cc d kin cho to ra mi lin h gia cc n s (cc phng trnh),
gii h phng trnh, tm ra gi tr ca n s cn tm. Phng php tng t cng c th
p dng cho cc bi ton hnh hc tnh ton (chng hn bi ton gii tam gic, t gic),
cc bi ton m (phng php dy s ph).
Trong bi ny, chng ta cp ti phng php lp phng trnh, h phng trnh gii
cc bi ton phng trnh hm. tng chung cng l tm mt gi tr f(x) hoc f(a)
no , ta s dng phng trnh hm tm ra mi lin kt gia cc i lng, ni cch
khc, to ra cc phng trnh s. Gii cc phng trnh s ny, ta c th tm ra f(x) hoc
f(a) vi a l mt gi tr no .
1. Gii phng trnh hm da vo tnh xon ca hm s
Hm s (x) c gi l xon i vi php hp nu tn ti s nguyn dng k sao cho
k(x) = x vi mi x. y 1(x)=(x), 2(x)=((x)), , k(x) = (k-1(x)). Ta c th
s dng tnh xon ca mt s hm s gii mt s dng phng trnh hm vi mt bin
t do. Chng ta bt u t nhng v d n gin:
Bi ton 1. Tm tt c cc hm s f: R \ {0}  R tho mn iu kin
1
( x 1) f ( x ) + f = x 2
x

(1) vi mi x 0

Gi
i. Trong (1), thay x bng 1/x, ta c
1
1 1
1 f + f ( x ) = 2
x
x x

(2)

Gii h (1), (2), coi f(x) v f(1/x) l cc n s, ta c


f ( x) =

x4 x3 +1
.
x3 x2 + x

50 | Trn Nam Dng 6/2010

Vietnamese IMO Team Training Camp 2010


Trong v d trn, ta cn tm f(x). Ta coi f(x) l n s chnh, cn f(1/x) l mt n ph.
Phng trnh (1) cho chng ta mt s lin kt gia n s chnh v n ph. Thay x bng
1/x, ta hy vng s tm ra c mi lin kt mi. Trong trng hp ny, chnh l
phng trnh (2). V phng trnh (2) khng to ra n s mi, tc l s phng trnh
bng s n s, cho gii tm c f(x) nn ta dng li. Trong v d tip theo, chng ta
s thy qu trnh to cc mi lin kt c th to ra cc n s mi.
Bi ton 2. Tm tt c cc hm s f: R \ {0, 1}  R tho mn iu kin
2
1 + x x +1
f ( x) + ( x + 1) f
=
1 x x 1

(1)

1+ x
. (1) cho ta mt phng trnh kt
1 x
1+ x
th c
ni 2 n s ny. Ta tm cc mi lin kt khc. Trong (1) ta thay x bng
1 x
2
1 + x 1 + x + 1
1+

1+ x 1 + x 1 x 1 x
f
+ 1 f
=
+
1+ x
1 x 1 x 1 1+ x
1

1 x
1 x
2
x2 +1
1+ x
1
f
f =
(2)
+
x( x 1)
1 x 1 x x
1
Nh vy, trong (2) li xut hin mt n s ph mi, l f . Nu dng li y th
x

Gii. n s y ang l f(x) v n s ph l f

s n s nhiu hn s phng trnh v ta s khng tm c f(x). Trong (1), tip tc thay


x bng -1/x th ta c
1
1

+1
1

x2 +1
1 1
1 1 1 x
x = x2
f + + 1 f
f + + 1 f
=

(3) Ta

x( x + 1)
x x 1 + 1 1 1
x x 1+ x

x
x

1 x
tip tc nhn c mt n s ph mi, l f
. . Trong (1), ta li thay x bng
1+ x
1 x
th c

1+ x
2
1 x 1 x + 1
1

1 x 1 x 1+ x 1+ x
f
+ 1 f
=
+
1 x
1+ x 1 + x 1+ 1 x

1+ x
1+ x
x2 +1
1 x 2x
( 4)
f
f (x ) =

x +1
1+ x 1+ x

51 | Trn Nam Dng 6/2010

Vietnamese IMO Team Training Camp 2010


n y th n s mi khng xut hin, nh vy ta c 4 phng trnh (1), (2), (3),
1+ x
,
1 x

(4) vi 4 n s l f ( x), f

1
f ,
x

1 x
f
. Gii h ny nh mt h phng trnh
1+ x

bc nht 4 n, ta tm c
x2 +1
.
f ( x) =
x( x 1)

Quan st li gii ca hai bi ton trn, ta thy rng phng php tng t c th p dng
cho bi ton tm tt c cc hm f(x) tho mn phng trnh
f(x) + a(x)f((x)) = b(x) (1)
Trong a(x), b(x) l cc hm s cho, cn (x) l mt hm cho tho mn tnh cht
k(x) = x vi mi x. S k nh nht tho mn iu kin k(x) = x c gi l bc ca hm
s (x).
Cc v d thng gp v hm (x) chnh l (x) = -x, (x) = 1/x. Hm ( x) =
trong v d 2 l mt hm bc 4. C th rng kim tra hm s ( x) =

3+x
1 3x

1+ x
1 x

c bc l 3.

Ch rng a s cc hm s l khng xon. Chng hn trong cc hm a thc, ch c


hm s f(x) = x v f(x) = a x l xon. Mt lp hm phn thc xon c m t phn
bi tp.
Ngay c vi phng trnh hm cha cc hm xon, ta cng c th gp kh khn khi do
tnh i xng, ta khng to ra s phng trnh gii (c th xy ra trng hp c
mt phng trnh no l h qu ca cc phng trnh khc). V d:
Bi ton 3. Tm tt c cc hm s f: R  R tho mn iu kin
f(x) + f(-x) = x2
vi mi x thuc R.
Gii. Trong ng thc f(x) + f(-x) = x2 (1) thay x bng x th ta c
f(-x) + f(x) = (-x)2
(2)
Nh vy khng xut hin n s mi nhng cng lc , (2) cng khng phi l phng
trnh mi, m hon ton ging nh (1). Nh vy, ta khng th gii c h (1), (2) ra
gi tr duy nht ca f(x).
Cng tng t nh trong trng hp h phng trnh tuyn tnh vi s n s nhiu hn
s phng trnh trng hp ny h ca chng ta, hay ni cch khc, phng trnh (1) s
c v s nghim. C th trong bi ton ny, t g(x) = f(x) x2/2 th ta c g(x) + g(-x)
= 0, suy ra g(x) l mt hm s l. Ngc li, nu f(x) = x2/2 + g(x) vi g(x) l mt hm s
l bt k th r rng ta c
f(x) + f(-x) = x2/2 + g(x) + x2/2 + g(-x) = x2.
52 | Trn Nam Dng 6/2010

Vietnamese IMO Team Training Camp 2010

Vy tt c cc hm s f(x) tho mn iu kin bi l f(x) = x2/2 + g(x), trong g(x) l


mt hm s l bt k xc nh trn R.
Bi tp
1. Tm tt c cc hm s f: R  R sao cho vi mi x {-1, 1} ta c
x 3
3+ x
f
+ f
=x
x +1
1 x

2. Gi s a 0. Tm hm s f(x) bit rng


a2
= x
f ( x) + f
ax

3. Tm hm s f(x) bit rng


x
3 x 2 13x 4
2f
3f
=
2
2 x + 1 2 x 3x
x 1

4. a) Chng minh rng hm s ( x) =

1 + 3x
3x

l xon

b) Tm tt c cc gi tr a sao cho hm s ( x) =

a+x
l xon.
1 ax

5. Tm tt c cc hm s f(x): R  R sao cho vi mi x khc 0 ta c


f(x) + f(1/x) = 2
6. Tm tt c cc hm s f: R \ {0, 1}  R tho mn phng trnh
1 2(1 2 x)
f ( x) + f
=
1 x x(1 x)

vi mi x thuc min xc nh ca f.
2. Gii phng trnh hm bng cch lp phng trnh
Vi nhng phng trnh hm c 2 (hoc nhiu hn) phng trnh iu kin, ta c th tm
cch kt hp cc phng trnh tm ra f(x). Phng php c bn vn l to ra cc
mi lin kt, hay cc phng trnh bng cch tnh mt gi tr bng hai cch khc nhau.
Bi ton 4. Tm tt c cc hm s f: R  R tho mn iu kin
i) f(-x) = -f(x) vi mi x thuc R;
ii) f(x+1) = f(x) + 1 vi mi x thuc R;
53 | Trn Nam Dng 6/2010

Vietnamese IMO Team Training Camp 2010


1
x

iii) f =

f ( x)
vi mi x khc 0.
x2

Gii. Tt c cc iu kin u trn mt bin x. Trong trng hp ny, ta c th dng mt


cht khi nim v th hiu con ng i n li gii. Ta xem cc s thc nh cc
nh ca mt th. nh x s c ni vi cc nh x+1, -x, 1/x. Cc iu kin bi s
cho chng ta cc mi lin h gia gi tr ca hm s ti cc nh c ni bi mt cnh.
Nu chng ta tm c mt chu trnh th mt cch t nhin, chng ta s c 1 phng trnh
( trnh hm s c hai gi tr khc nhau).
Ta th tm mt chu trnh nh vy
x x +1

1
1
1
x
x +1
1
1

1
=

=1+ x
x +1
x +1
x +1 x +1
x
x
x

t y = f(x) th t chu trnh trn, ta ln lt c


y +1
1
y +1
y +1
1

x
f ( x + 1) = y + 1, f
, f
, f
=
=
=1
2
2
( x + 1)
( x + 1) 2
x + 1 ( x + 1)
x +1
x +1
y +1
1
( x + 1) 2 x 2 + 2 x y 1 2 x y
x + 1
f
=
, f =
, f ( x) = 2 x y
=
2
x2
x2
x
x
x

x + 1

T suy ra 2x y = y, tc l y = x. Vy f(x) = x.
Trong l lun trn, ta cn n iu kin x khc 0 v -1. Tuy nhin t iu kin f(x+1) =
f(x) + 1 ta d dng suy ra f(0) = 0 v f(-1) = 1. Vy f(x) = x l tt cc nghim ca bi
ton.
Bi ton 5. Tm tt c cc hm s f: R  R tho mn iu kin
f(x2-y) = xf(x) f(y) vi mi x, y thuc R
Gii. Thay x = y = 0 vo phng trnh hm, ta c f(0) = - f(0), suy ra f(0) = 0. Thay y
= 0 v phng trnh hm, ta c
f(x2) = xf(x)
(1)
T suy ra
f(x2y) = f(x2) f(y)
Thay x = 0, ta c f(y) = f(y). Thay y bng y, ta c
f(x2+y) = f(x2) f(y) = f(x2) + f(y) vi mi x, y.
T , kt hp vi tnh cht hm l, ta suy ra f(x+y) = f(x) + f(y) vi mi x, y.
By gi ta c f((x+1)2) mt mt c th tnh theo cng thc (1), tc l bng (x+
1)f(x+1) = (x+1)(f(x)+f(1)). Mt khc, ta c th khai trin
f((x+1)2) = f(x2+2x+1) = f(x2) + 2f(x) + f(1) = xf(x) + 2f(x) + f(1).
T ta c phng trnh (x+1)(f(x)+f(1)) = xf(x) + 2f(x) + f(1), suy ra f(x) = f(1)x.
t f(1) = a, ta c f(x) = ax. Th li vo phng trnh ta thy nghim ng.
54 | Trn Nam Dng 6/2010

Vietnamese IMO Team Training Camp 2010

Vy f(x) = ax vi a R l tt c cc nghim ca bi ton.


Phng php to ra cc mi lin kt cng c th p dng hiu qu trong cc bi ton
phng trnh hm trn Q, N, Z. Ta xem xt mt s v d
Bi ton 6. Tm tt c cc hm s f : Q+  Q+ tho mn cc iu kin
i)
f(x+1) = f(x) + 1 vi mi x thuc Q+;
ii)
f(x2) = f2(x) vi mi x thuc Q+.
Gii. T iu kin i) ta d dng suy ra f(n) = n vi mi n thuc Z v f(r+n) = f(r) + n vi
mi r thuc Q v n thuc Z. By gi ta tnh f(r) vi r =

p
. tng ta s tnh f((r+q)2)
q

theo f(r) bng hai cch.


Trc ht
f((r+q)2) = f2(r+q) = (f(r) + q)2 (1)
Mt khc
f((r+q)2) = f(r2+2p+q2) = f(r2) + 2p + q2 = f2(r) + 2p + q2 (2)
T (1) v (2) ta suy ra f2(r) + 2qf(r) + q2 = f2(r) + 2p + q2 => f(r) = p/q = r.
Vy f(r) = r vi mi r thuc Q.
Bi ton 7. Tm tt c cc hm s f: N  N sao cho
f(m2+n2) = f2(m) + f2(n) vi mi m, n thuc N
Gii. Cho m = n = 0, ta c f(0) = 2f2(0), suy ra f(0) = 0. Cho m = 1, n = 0, ta c f(1)
= 0 hoc f(1) = 1. Ta xt trng hp f(1) = 1, trng hp f(1) = 0 xt tng t. Vi f(1) =
1, ta ln lt tnh c
f(2) = f(12+12) = f2(1) + f2(1) = 2
f(4) = f(22+02) = f2(2) + f2(0) = 4
f(5) = f(22+12) = f2(2) + f2(1) = 5
Nhng lm sao tnh, chng hn f(3)? R rng f(3) khng th tnh c theo s trn
c, v 3 khng biu din c di dng tng ca hai bnh phng.
Ta nh li mt bi ton lp 3. C 1 ci cn a vi 2 qu cn 1kg, 5kg v 1 bao ng
nng 10kg. Hy cn ra 7kg ng bng 1 ln cn. R rng, vi cch cn thng thng
th ta ch cn c 1kg ng, 4kg ng (5-1), 5 kg ng v 6kg ng. Tuy nhin,
nu tinh 1 cht, ta c th c phng n cn c 7kg ng nh sau: t vo a bn
tri qu cn 1kg v 10kg ng, a bn phi l qu cn 5kg, sau chuyn dn ng
t bn tri sang bn phi sao cho cn cn bng, khi s ng cn li a bn phi
l 7kg !
By gi ta cng th thut tng vi bi ton ny. Ta khng tnh c trc tip f(3) nhng
ta li c f2(5) = f(25) = f(32+42) = f2(3) + f2(4). T ta c f(3) = 3.
Tng t nh vy ta c th tnh c f(6) nh vo ng thc 62 + 82 = 102, trong f(8)
= f(22+22) = 2f2(2) = 8, f(10) = f(32+12) = f2(3) + f2(1) = 10.
55 | Trn Nam Dng 6/2010

Vietnamese IMO Team Training Camp 2010


Tip tc, tnh f(7), ta 72 + 1 = 50 = 52 + 52, t f(7) = 7. Cng nh th, do 112 +
22 = 102 + 52 nn ta suy ra f(11) = 11.
Cch lm ny c th tng qut ho nh th no? tng l nu m2 + n2 = p2 + q2 (1) th
f2(m) + f2(n) = f2(p) + f2(q). Do nu ta tnh c f(n), f(p), f(q) th f(m) cng s tnh
c.
Lm th no c c nhng ng thc dng (1) dng tng qut, cho php ta chng
minh f(n) = n vi mi n bng quy np? Ch rng (1) c th vit li thnh (m-p)(m+p) =
(q-n)(q+n) = N. Do nu chn nhng s N c 2 cch phn tch thnh tch ca nhng s
c cng tnh chn l, ta s tm c nghim cho (1). Chn N = 8k = 2.4k = 4.2k v N =
16k = 4.4k = 8.2k, ta c h
m p = 2, m+p = 4k, q n = 4, q + n = 2k
v
m p = 4, m+p = 4k, q n = 8, q + n = 2k
T c cc hng ng thc tng ng
(2k+1)2 + (k-2)2 = (2k-1)2 + (k+2)2
v
(2k+2)2 + (k-4)2 = (2k-2)2 + (k+4)2
T hai ng thc ny, vi ch l ta chng minh c f(n) = n vi n = 0, 1, 2, 3, 4, 5,
6, ta d dng chng minh bng quy np c rng f(n) = n vi mi n N.
Trng hp f(1) = 0, cng bng cch l lun nu trn ta suy ra f(n) = 0 vi mi n thuc
N.
Bi tp.
1. Tm tt c cc hm s f: Q  Q tho mn cc iu kin
i) f(x+1) = f(x) + 1 vi mi x thuc Q;
ii) f(x3) = f3(x) vi mi x thuc Q;
2. Tm tt c cc hm f: R \ {0}  R tho mn ng thi cc iu kin
i) f(1) = 1;
1

= f + f
ii) f
x
x+ y
y
iii) (x+y)f(x+y) = xyf(x)f(y)
vi mi x, y m xy(x+y) 0.

3. Tm tt c cc hm s f: R  R tho mn
f(x5 y5) = x2f(x3) y2f(y3) vi mi x, y thuc R.
4. Tm tt c cc hm s f: Z  Z tho mn iu kin
56 | Trn Nam Dng 6/2010

Vietnamese IMO Team Training Camp 2010


f(a3+b3+c3) = f3(a) + f3(b) + f3(c)
vi mi a, b, c thuc Z.
5. Cho hm s f: R  R tho mn iu kin
i) f(x2) = f2(x) vi mi x thuc R;
ii) f(x+1) = f(x) + 1 vi mi x thuc R.
Chng minh rng f(x) = x.
3. Tnh gi tr ca hm s ti mt im c bit bng cch lp phng trnh
Trong phn trn, chng ta c th thy rng phng php lp phng trnh c th gip
chng ta tm c f(x) vi mi x, ni cch khc l gii c phng trnh hm. Trong
tnh hung tng qut, khng phi lc no ta cng thc hin c iu . Mi lin kt c
th ch tn ti i vi mt s cc gi tr c th. Ta xt mt v d:
Bi ton 8. Cho f l hm s khng gim xc nh trn on [0, 1], tho mn ng thi
cc iu kin
1) f(0) = 0
2) f(1x) = 1 f(x) x [0, 1];
3) f(x/3) = f(x)/2 x [0, 1].
Hy tnh f(1/13), f(1/7).
Gii. Tng t nh bi ton 4, ta thit lp th c nh l cc s thc thuc [0, 1] v
x c ni vi 1 x v x/3 (l cc i lng c th tnh c nu bit f(x)). Ta thit lp
chui
1
1 12
4
4
9
3
1
1 =

13
13 13
13
13 13 13 13
1
Nh vy, nu t f = y th ta ln lt c
13
1 y 1+ y 3 1+ y 1 1+ y
12
4 1 y 9
f = 1 y, f =
, f = 1
=
, f =
, f =
2
2
2
4
8
13
13
13
13
13
1+ y
1
1 1
Nh vy ta phi c y =
, t y = . Vy f = .
8
7
13 7

Lu rng li gii ni trn l rt c th cho s 1/13. V d vi 1/7 th chui nh trn


s khng ng c
1
6
2
5

7
7
7
7

Nh vy, cch gii trn y khng p dng cho 1/7. tm gi tr f(1/7), ta cn n cc


iu kin f khng gim v f(0) = 0 (cc iu kin ny khng cn n khi tnh f(1/13)!).
C th, t cc iu kin bi, ta suy ra
f(0) = 0, f(1) = 1, f(1/3) = f(1)/2 = 1/2, f(2/3) = 1 f(1/3) = 1/2
57 | Trn Nam Dng 6/2010

Vietnamese IMO Team Training Camp 2010


T , do hm f khng gim nn ta suy ra f(x) = 1/2 vi mi x [1/3, 2/3]. p dng tnh
cht 3, ta suy ra f(x) = 1/4 vi mi x x [1/9, 2/9]. M 1/9 < 1/7 < 2/9 nn t y ta suy
ra f(1/7) = 1/4.
Trong cc bi ton phng trnh hm trn tp s nguyn, cc mi lin h dng bt ng
thc gia cc s cng c th l cha kho tm c gi tr ca hm s ti mt im.
Bi ton 9. Cho hm s f: N*  N* tho mn ng thi cc iu kin sau
i)
f(2) = 4 ;
ii)
f(mn) = f(m).f(n) vi mi m, n thuc N* ;
iii)
f(m) > f(n) vi mi m > n.
Hy tm f(3).
Gii. D thy hm s f(n) = n2 tho mn c ba iu kin. Mt cch t nhin, ta ngh n
vic chng minh f(3) = 9.
Thng tin duy nht m ta c th c c t hai iu kin u l f(2k) = 4k. R rng, nu
khng c iu kin iii) th f(3) c th nhn gi tr tu , do f(3) khng tnh c.
Vi iu kin iii), ta c 4 = f(2) < f(3) < f(4) = 16, suy ra f(3) ch c th nhn gi tr trong
{5, 6, , 15}.

Tuy nhin, ch cn xt thm bt ng thc 8 < 9 l ta c th suy ra 64 = f(8) < f(9) =


f2(3), suy ra f(3) > 8. V bt ng thc 27 < 32 cho ta f3(3) = f(27) < f(32) = f5(2) =
1024, suy ra f(3) 10.
Nh vy, f(3) ch cn c th nhn 1 trong 2 gi tr 9 hoc 10.
Ta tm cch loi gi tr 10 bng 1 bt ng thc st hn. Tm kim trong cc lu tha ca
2 v 3, ta tm c cp bt ng thc 35 = 243 < 256 = 28. Nu f(3) = 10 th t bt ng
thc trn ta s suy ra 105 < 48  100000 < 65536, mu thun.
Vy ch cn trng hp f(3) = 9. Kt hp vi nhn xt ban u l hm s f(n) = n2 tho
mn tt c cc iu kin ca bi, ta kt lun f(3) = 9.
Ta c th s dng tng trn tnh f(n) vi n bt k, c th l chng minh f(n) = n2
vi mi n. Ni chung, trong mt phng trnh hm tng qut th vic tm gi tr hm s
ti tng im nh vy c th s gp kh khn. Tuy nhin, c th th
y rng, trong mt s phng trnh hm th vic tm cc gi tr c bit nh f(0), f(1), f(-1)
ng vai tr then cht trong li gii ca bi ton. Di y ta xem xt mt s v d.
Bi ton 10. Tm tt c cc hm s f: R  R tho mn iu kin
58 | Trn Nam Dng 6/2010

Vietnamese IMO Team Training Camp 2010


f(xy) + f(x+y) = f(x)f(y) + f(x) + f(y)

(1)

Gii. Phng trnh trn l tng ca hai phng trnh


(2) f(x+y) = f(x) + f(y)
(3) f(xy) = f(x)f(y)
Ch l theo l thuyt v phng trnh hm Cauchy th nu hm s f: R  R tho mn
ng thi hai phng trnh (2) v (3) th f(x) = x hoc f(x) 0. Do vy, ta ch cn chng
minh l t (1) suy ra (2) v (3) l xong.
tch c cc phng trnh trong (1), ta quan tm n tnh chn, l ca f. Chng hn
nu f l hm s l th thay y bng y, ta s thu c
f(-xy) + f(x-y) = f(x)f(-y) + f(x) + f(-y) (4)
Cng (1) v (4), ch f l hm l, ta s thu c
f(x+y) + f(x-y) = 2f(x)
T phng trnh ny v f(0) = 0 (suy ra t (1) bng cch cho x = y = 0), ta d dng suy ra
f(x+y) = f(x) + f(y) vi mi x, y, tc l (1) tch c.
Quay tr li vi bi ton. Nh vy, ta quan tm n tnh chn l ca hm s f.
Mt cch t nhin, ta thay y = -1 trong (1) th c
f(-x) + f(x-1) = f(x)f(-1) + f(x) + f(-1)
(5)
Nu tm c f(-1) th c th kho st c tnh chn l ca f. Ta tm cch tnh

f(-1).

Ch rng trong phng trnh (5) c n 3 n s l f(-x), f(x-1), f(x). iu ny khng


cho php chng ta thit lp dy truy hi tnh cc gi tr f. Mt la chn khc l thay y
= 1 vo (1) c
f(x) + f(x+1) = f(x)f(1) + f(x) + f(1)
Suy ra
f(x+1) = f(1)f(x) + f(1)
(6)
Nh vy, theo ngn ng th dng trong cc bi ton 4, 8, ta tm c mi lin
kt x  x+1.
T y, nu t f(1) = a th ta ln lt tnh c f(2) = a2 + a, f(3) = a3 + a2 + a, f(4) = a4
+ a3 + a2 + a.
Nu tip tc dy ny th s tnh c f(n) theo f(1). Tuy nhin, iu ta cn tm l f(1). V
th, ta cn tm mt phng trnh khc vi cng thc truy hi ni trn. iu ny c th c
c ngay nu ta thay x = y = 2 vo (1). Khi
f(4) + f(4) = f2(2) + 2f(2)
Thay cc gi tr ca f(4) v f(2) tnh theo a trn vo, ta c phng trnh
2(a4+a3+a2+a) = (a2+a)2 + 2(a2+a)

a4 = a2
T suy ra a ch c th nhn cc gi tr a = 0, a = -1, a = 1.
59 | Trn Nam Dng 6/2010

Vietnamese IMO Team Training Camp 2010


+ Nu a = 0 th t (6) ta suy ra f(x) = 0 vi mi x. y l mt nghim ca phng trnh
hm.
+ Nu a = 1 th t (6) ta suy ra f(x+1) = f(x) 1, tc l f(x) + f(x+1) = 1 vi mi x.
T y, thay x = 1, ch l f(0) = 0, ta c f(1) = 1. Thay vo (5), ta c f(x) +
f(x1) = 1. M f(x1) + f(x) = 1 nn t y suy ra f(x) = f(x), tc l f l hm chn.
By gi trong (1), ta thay y bng y th c
f(-xy) + f(x-y) = f(x)f(-y) + f(x) + f(-y) (4)
Tr (1) cho (4), ch l f l hm chn, ta suy ra f(x+y) f(xy) = 0 vi mi x, y. T y
suy ra f l hm hng. iu ny mu thun v f(0) = 0, f(1) = 1. Vy trng hp ny
khng xy ra.
+ Nu a = 1 th t (6) ta suy ra f(x+1) = f(x) + 1 (7). Thay x = 1, ta c f(1) = 1.
Thay vo (5), ta c f(x) + f(x1) = 1. Li c t (7) f(x1) = f(x) 1 nn t y ta suy
ra f(x) = f(x). Nh vy f l hm l. L lun nh phn tch ban u, ta c f(x) tho
mn h phng trnh (2), (3). p dng l thuyt phng trnh hm Cauchy, ta tm c
f(x) = x.
Bi ton 11. Tm tt c cc hm s f: R  R tho mn iu kin
f(x2 + f(y)) = f2(x) + y
(1)
Gii. Vn mu cht trong bi ton ny l chng minh c f(0) = 0. Tht vy, nu ta
chng minh c f(0) = 0 th thay y = 0 vo phng trnh hm, ta c
f(x2) = f2(x)
(2)
Thay x = 0 vo (1), ta c
f(f(y)) = y
(3)
Thay y bng f(y) trong (1), ch rng f(f(y)) = y v f2(x) = f(x2), ta c f(x2+y)
=
f(x2) + f(y) vi mi x, y thuc R
T y d dng chng minh c rng
f(x+y) = f(x) + f(y)
(4)
vi mi x, y thuc R.
Nh vy f tho mn phng trnh hm Cauchy. Ngoi ra, do (2) nn ta c f(x) 0 vi
mi x 0. Kt hp vi (4), ta suy ra f l hm s tng. Nh vy, theo l thuyt c bn v
phng trnh hm Cauchy, f(x) = ax vi mi x, trong a 0. Thay vo phng trnh
hm, ta c a(x2+ay) = a2x + y vi mi x, y, suy ra a = 1. Vy f(x) = x l nghim duy
nht ca phng trnh hm.
Tt cc l lun trn y u da trn gi thit l ta chng minh c f(0) = 0. By gi
ta s chng minh iu ny.
t f(0) = a. Thay y = 0 vo (1), ta c
f(x2+a) = f2(x)
(5)
Thay x = 0 vo (5), ta c f(a) = a2
Thay x = 0 vo (1), ta c
60 | Trn Nam Dng 6/2010

Vietnamese IMO Team Training Camp 2010


f(f(y)) = a2 + y
(6)
2 2
By gi ta s tnh f(a +f (1)) bng hai cch. Mt mt
f(a2+f2(1)) = f(f2(1) + f(a)) = (f(f(1)))2 + a = (a2+1)2 + a
Mt khc
f(a2+f2(1)) = f(a2+f(1+a)) = (f(a))2 + 1 + a = a4 + a + 1
T y ta suy ra a4 + 2a2 + 1 + a = a4 + a + 1 suy ra a = 0. Tc l f(0) = 0. Li gii hon
tt.
Bi tp
1. Cho f l hm s khng gim xc nh trn on [0, 1], tho mn ng thi cc iu
kin
i) f(0) = 0
ii) f(1x) = 1 f(x) x [0, 1];
iii) f(x/3) = f(x)/2 x [0, 1].
a) Hy tnh f(18/1991);
b) Hy tnh f(1/n) vi n {1, 2, , 20}
c)* Chng minh rng tn ti duy nht mt hm s f tho mn ng thi cc iu kin
nu trn.
2. Tm tt c cc hm s f: N*  N* tho mn cc iu kin
i) f(2) = 2;
ii) f(mn) = f(m).f(n) vi mi (m, n) = 1;
iii) f(m) > f(n) vi mi m > n.
3. Tm tt c cc gi tr k thuc N* sao cho tn ti hm s f: N*  N* tho mn cc iu
kin
i) f(2) = k;
ii) f(mn) = f(m).f(n) vi mi m, n;
iii) f(m) > f(n) vi mi m > n.
4. Tm tt c cc hm s f: R  R tho mn iu kin
f(xy) + f(x) + f(y) = f(x+y) + f(x)f(y) vi mi x, y thuc R.
5. Tm tt c cc hm s f : R  R tho mn iu kin
(f(x+y))2 = f(x)f(x+2y) + yf(y) vi mi x, y thuc R.
6. Tm tt c cc hm s f: R  R tho mn iu kin
f(x2 + y + f(y)) = f2(x) + 2y
vi mi x, y thuc R.

61 | Trn Nam Dng 6/2010

Vietnamese IMO Team Training Camp 2010


Ti liu tham kho
[1] Nguyn Vn Mu, Phng trnh hm, Nh xut bn Gio dc 2001
[2] Nguyn Trng Tun, Bi ton hm s qua cc k thi Olympic, Nh xut bn Gio dc
2005.
[3] Phan c Chnh, L nh Thnh, Phm Tn Dng, Tuyn tp cc bi ton s cp,
Tp 1, i s, NXB i hc v THCN 1977
[4] Phan Huy Khi, Cc bi ton v hm s, Nh xut bn Gio dc 2007
[5] B.J.Venkatachala, Functional Equations A Problem Solving Approach, PRISM
2002.
[6] Pierre Bornsztein, Mobinool Omarjee, Cours Equations fonctionelles, Electronic
Edition 2003
[7] Titu Andreescu, Iurie Boreico, Functional Equations, Electronic Edition 2007

62 | Trn Nam Dng 6/2010

Vietnamese IMO Team Training Camp 2010

Ti u t hp I: Cc bi ton ti u v h cc tp hp
Gil Kalai
Bi vit ny gii thiu v cc bi ton ti u lin quan n h cc tp hp. y l mt
trong nhng bi ging ca Gil Kalai ti seminar Cc khi nim c bn ti H TH
Hungari do David Kazhdan khi xng.

Paul Erds
1. Ba bi ton m u
Chng ta s bt u bng ba bi ton kh ging nhau, t rt d n rt kh.
Bi ton I: Cho N = {1,2, , n }. Hi kch thc ln nht ca h
cc tp con ca
tho mn iu kin hai tp hp bt k thuc c phn giao khc rng? (Mt h nh vy
c gi l h giao nhau)
Tr li I: Kch thc ln nht l
. Chng ta c th t c iu ny bng cc chn
tt c cc tp con cha phn t 1. Chng ta khng th t c con s ln hn, bi t
mi cp gm mt tp hp v phn b ca n, ta ch c th chn mt tp hp vo h cc
tp hp ca chng ta.
Bi ton II: Tm kch thc ln nht ca h
bt k thuc c hp khc ?.

cc tp con ca

63 | Trn Nam Dng 6/2010

sao cho hai tp hp

Vietnamese IMO Team Training Camp 2010


Cu tr li l hin nhin v bi ton II chng qua l bi ton I. Ch cn chuyn qua phn
b. V ta s c p s y nh bi ton I. Ta pht biu bi ton II khc.
Bi ton II mi: Tm kch thc ln nht ca h
cc tp con ca sao cho vi hai
tp hp bt k
thuc ta c giao ca chng khc rng v hp ca chng khc .
Mt v d ca mt h nh vy l tp hp tt c cc tp hp cha phn t 1 nhng khng
cha phn t 2. H ny c
tp hp. Phi mt vi nm sau khi Erds xut bi
ton ny, Kleitman mi chng minh c l khng c h no ln hn vi tnh cht ny.
Bi ton III (Gi thuyt Erds-Sos) Cho l h cc th vi N l tp cc nh. Gi sa
rng hai th bt k ca h c chung mt tam gic. Hi c kch thc ln nht bng
bao nhiu?
Tng s cc th vi n nh l
. (Ch : ta m cc th m khng tnh n s
ng cu ca chng.) Mt v d n gin ca h tp hp vi tnh cht cho l tt c cc
tp hp cha mt tam gic c nh no . V d tt c cc th cha cc cnh
{1,2},{1,3},{2,3}. H ny cha 1/8 ca tt c cc th. Tn ti hay khng mt h ln
hn cc th vi tnh cht cho? Erds v Sos a ra gi thuyt l cu tr li l khng
chng ta khng th tm c h ln hn. Gi thuyt ny hin nay vn l mt vn
m.

Vera Sos
2. Hai nh l c bn v h cc tp con vi cc iu kin giao cho trc.
nh l Erds-Ko-Rado: Mt h giao nhau cc k-tp con ca
ti a
tp hp.
64 | Trn Nam Dng 6/2010

, trong

cha

Vietnamese IMO Team Training Camp 2010


nh l Fisher; deBruijn-Erds: H cc tp con ca sao cho hai tp hp khc nhau
bt k ca h c ng mt phn t chung c nhiu nht n phn t.
(Erds v deBruijn kt lun rng n im khng thng hng trn mt phng xc nh t
nht n ng thng. Hy th suy ra iu ny t nh l trn!)
Tt c cc k-tp con ca N cha phn t 1 cho v d v du bng cho nh l Erdos-KoRado. Vi nh l Erds deBruijn ta ly h {{1} , {1,2} {1,3} {1,n}} hoc thay tp
hp u tin bi {2,3,,n}, hoc ly mt mt phng x nh hu hn.

Mt phng Fano cc im x nh hu hn bc 2.
3. Php chng minh nh l deBruijn Erds bng i s tuyn tnh.
Php chng minh nh l Fisher; de Bruijn-Erds c th trnh by nh sau: Gi s rng
c m tp hp
trong h. Xt ma trn lin kt ca h: Phn t (i,j) ca ma
trn ny bng 1 nu i thuc .
S kin ct yu y l cc ct
.
y suy ra

ca ma trn lin kt l c lp tuyn tnh, t

Lm th no c th chng minh rng cc ct l c lp tuyn tnh? Trc ht, chng ta


gi s rng c t nht 2 tp hp. Khi ta vit
, v tnh tch trong
.
Ta lu rng nu i v j khc nhau th

Nh vy

.
65 | Trn Nam Dng 6/2010

Vietnamese IMO Team Training Camp 2010


Nh vy tch ny ch c th bng 0 khi tt c cc h s

u bng 0.

Chng minh ny l mt v d ca cc l lun v chiu trong t hp.


4. nh l Sperner
nh l Sperner (1927) khng nh rng kch thc ln nht ca h
l mt i xch i vi quan h bao hm l h s nh thc
chng minh n gin v p cho nh l Sperner:

cc tp con ca N

. Lubell tm c mt php

l mt i xch nh vy v gi s rng n c tp hp k phn t. Ta tnh cc cp


trong =
l mt hon v ca {1,2, ,n} v S l tp hp
thuc h c khi u l , c th l S={
} vi k no . Vi mi hon
v ta tm c nhiu nht mt khi u S trong h (do iu kin i xch). Nu S l
mt tp hp k phn t, ta c th tm c ng k! (n-k)! hon v vi S l khi u. Kt
hay ni cch khc
.
hp hai s kin ny, ta c
Bt ng thc ny (c gi l bt ng thc LYM) suy ra kt qu cn chng minh.
Gi

Bella Bollobas, mt trong nhng ngi pht hin ra bt ng thc LYM.


nh l Erds-Ko-Rado c mt php chng minh tng t vi chng minh trn.
tng l tnh cch cp
quanh

trong l mt tp hp trong h,
l mt khong lin tc i vi .

l hon v vng

Mt mt ta c (n-1)! hon v vng quanh v d dng thy rng vi mi mt hon v nh


vy, ta c th chn c nhiu nht k khong i mt giao nhau. Mt khc, vi mi tp
hp S c k!(n-k)! hon v vng quanh m trong S l mt khong lin tc.
Nh vy

k! (n-k)!

v iu ny cho chng ta nh l Erds Ko Rado.

Quay tr li mt cht v cu d dng nhn thy. Phn ny s dng iu kin


.
Mt cch l lun cho phn ny nh sau: xt khong J m phn t tn cng bn tri l nm
bn tri nht v ch rng c k khong giao vi J m phn t tn cng bn tri nm
66 | Trn Nam Dng 6/2010

Vietnamese IMO Team Training Camp 2010


bn phi z. Mt cch khc l xt mt khong J bo c di k v ch rng 2k-2
khong c giao vi khong ny c chia thnh k-1 cp m mi cp cha hai khong
khng giao nhau.
5. nh l Turan v bi ton Turan
Trng hp c bit ca nh l Turan cho th khng cha tam gic c chng minh
bi Mantel vo nm 1907.
S ln nht cc cnh (k hiu l
) ca mt th n nh v khng ch tam gic t
c th hai phe y n nh vi kch thc hai phe cng gn nhau cng tt (c th
l [n/2] v [(n+1)/2].
nh l Turan dng tng qut c chng minh vo nhng nm 40 ca th k trc.
S ln nht cc cnh (k hiu l
) ca th n nh khng cha th con y
(r+1) nh t c ti th r phe y vi n nh, trong kch thc ca cc phn
cng gn nhau cng tt.

Paul Turan
Chng minh nh l Turan: Thc s nh l Turan khng kh; gn nh cch tip cn
no cng c th thnh cng. Sau y l mt cch tip cn nh vy: n gin, ta xt
trng hp tam gic. Xt nh v vi bc ln nht v chia cc nh cn li ca th thnh
hai phn: A cc nh k vi v, B l cc nh cn li. By gi ch l cc nh thuc
A lp thnh mt tp hp c lp (tc l khng c cnh ni gia cc nh ca A). Vi mi
nh thuc B ta xo i tt c cc cnh cha nh ny v thay vo , ni nh ny vi tt
c cc nh thuc A. rng trong th mi, bc ca mi nh u khng nh hn
bc th ban u. V, hn na, th mi l th hai phe (trong A l mt phe).
Cui cng ta ch cn chng minh l vi th hai phe th s cnh l ln nht khi hai phn
c s nh cng gn nhau cng tt.
Sau y l mt chng minh khc. Xo i mt nh ca th G vi n+1 nh khng ch
. S cnh ca th cn li khng vt qu
. Thc hin iu ny i vi tt c cc
nh v ch rng mt mt cnh c tnh n-1 ln. Ta thu c rng s cnh trong G (v
67 | Trn Nam Dng 6/2010

Vietnamese IMO Team Training Camp 2010


ngha l
) khng vt qu phn nguyn trn ca
ny cho chng ta kt qu chnh xc ca bi ton.

. nh gi

Chng ta kt thc chuyn tham quan th v ny bng bi ton m Turan a ra vo nm


1940. Chng ta mun tm s phn t ln nht ca tp hp cc b ba lp t {1,2,,n}
khng cha mt t din, tc l khng cha bn b ban c dng
{a,b,c},{a,b,d),{a,c,d},{b,c,d}.
Nu c gi cha bit p s, hy th a ra d on ca mnh. Turan a ra mt gi
thuyt ca mnh v gi thuyt ny hin nay vn cn l mt vn m.
Trn Nam Dng dch v gii thiu (t web site ca Seminar: Ti u t hp I)
Mt s ghi ch thm ca dch gi:
i xch: Trong mt tp sp th t (partial order set), mt i xch l mt h cc phn t
i mt khng so snh c vi nhau.
LYM: Lubell, Yamamoto, Meshalkin l nhng ngi chng minh nh l ny mt
cch c lp. Bollobas l ngi th t cng tm ra kt qu ny mt cch c lp.

68 | Trn Nam Dng 6/2010

Vietnamese IMO Team Training Camp 2010

V k thi chn i tuyn Vit Nam tham d IMO 2010


Trn Nam Dng
K thi chn i tuyn Vit Nam tham d IMO 2010 (VTST 2010) c t chc trong hai
ngy 17, 18/4 vi s tham gia ca 42 th sinh n t cc tnh thnh v cc trng H.
Thnh phn tham d TST nm nay c kh nhiu im c bit. Hai i c lc lng
hng hu nht tham d TST l Ngh An (6 em) v Nng (5 em). Cc n v ln u
tin c th sinh d TST l B Ra Vng Tu (2 em), HSP Tp HCM (1 em). Mt im
ni bt na l Ph Yn vi 3 hc sinh tham d TST. Bn cnh , c th nhn mnh s
vng mt ca mt s n v c truyn thng nh Hi Phng, Thanh Ha, H Vinh hay s
xung sc ca cc n v c s m khc nh Vnh Phc, PTNK.
Nh thng l, thi chn i tuyn nm nay c 6 bi. im c bit l nm nay ch c
1 bi hnh, 1 bi i s. Cc phn mn S hc v T hp c u i hn vi 2 bi.
Bi 1 v 4 c coi l cc bi d ca k thi. Bi 2 thuc loi trung bnh. Cc bi 5, 6
c nh gi l kh v bi 3 l rt kh. nh gi chung l nm nay khng kh bng
nm ngoi.
Cng tc chm thi c tin hnh ngay sau k thi v kt qu chn c 6 thnh
vin tham d IMO 2010.
Tri vi d on lc quan ca nhiu ngi, im thi nm nay khng cao hn nhiu so vi
nm ngoi, im cao nht i tuyn l 24 v im thp nht l 18. Nh vy, k thi nm
nay mt ln na li khng nh chn l: Nu lm 3 bi chc n th s lt vo i tuyn.
Kt qu ny tip tc cho thy im yu chung ca th sinh vn l hai mng S hc v T
hp, cng nh khu trnh by ca th sinh rt c vn . Sau k thi, kh nhiu th sinh
tuyn b mnh lm c 4 bi nhng kt qu thc t cho thy khng phi nh vy.
C mt s kin cho rng cc bi ton 5, 6 khng ph hp vi thi chn i tuyn v
em li li th cho nhng ai bit nh l Hall v nh l Lucas. Tuy nhin, theo kin
ca chng ti, mc k thi chn i tuyn, nu chng ta mun c mt i tuyn
mnh, sc tn cng cc bi ton 3, 6 ca IMO th cn phi nng tm kin thc cng
nh suy lun ca hc sinh n mc cc nh l ny. Ch l cc nh l ny rt s
cp v cha ng nhiu phng php t duy v l lun p v hay.
Cui cng, xinh chc mng cc th sinh vt qua k thi Vietnam TST va qua, chc
i tuyn Vit Nam c thnh tch xut sc ti IMO 2010!

69 | Trn Nam Dng 6/2010

Vietnamese IMO Team Training Camp 2010


CHN I TUYN VN D THI TON QUC T
Ngy 1, 17/4/2010
Thi gian lm bi: 240 pht.
BI 1. (6 im) Cho tam gic ABC khng vung ti A c trung tuyn AM. D l mt
im chy trn AM. Gi (O1), (O2) ln lt l cc ng trn i qua D v tip xc vi
BC ti B v C. CA ct (O2) ti Q. BA ct (O1) ti P.
a) Chng minh rng tip tuyn ti P ca (O1) v tip tuyn ti Q ca (O2) phi ct nhau.
Gi giao im ny l S.
b) Chng minh rng S lun chy trn mt ng c nh khi D chy trn AM.
BI 2. (6 im) Vi mi s n nguyn dng, xt tp sau
Tm tt c n sao cho khng tn ti a khc b thuc Tn sao cho a-b chia ht cho 110.
BI 3. (8 im) Hnh ch nht kch thc 1*2 c gi l hnh ch nht n. Hnh ch
nht 2*3 b di 2 gc cho nhau (tc cn c 4 ) gi l hnh ch nht kp. Ngi ta
ghp kht cc hnh ch nht n v hnh ch nht kp c bng 2008*2010. Tm s b
nht cc hnh ch nht n c th dng lt c nh trn.
Ngy 2, 18/4/2010
Thi gian lm bi: 240 pht.
BI 4. (6 im) Cho a, b, c l cc s thc dng tha mn iu kin
.
Chng minh rng
.
BI 5. (7 im) C n mi nc c k i din (n > k > 1). Ngi ta chia n.k ngi ny
thnh n nhm mi nhm c k ngi sao cho khng c 2 ngi cng nhm n t 1 nc.
Chng minh rng c th chn ra n ngi n t cc nhm khc nhau v n t cc nc
khc nhau.
BI 6. (7 im) Gi Sn l tng bnh phng cc h s trong khai trin ca (1+x)n. Chng
minh rng S2n + 1 khng chia ht cho 3.

70 | Trn Nam Dng 6/2010

Vietnamese IMO Team Training Camp 2010


Li gii v Nhn xt
Bi 1.
Ta c MB2 = MC2 nn M thuc trc ng phng ca (O1) v (O2). Suy ra DM l trc
ng phng ca 2 ng trn. Do A thuc trc ng phng ca 2 ng trn.
Suy ra AP.AB = AQ.AC => t gic BCPQ ni tip.
Gi tip tuyn ca (O1) l Px th xPB = PBC = PQA, suy ra Px tip xc vi (APQ)
hay (APQ) tip xc vi (O1). Tng t suy ra (APQ) tip xc vi c (O1) v (O2).
Tam gic APQ ng dng vi ACB nn APQ khng vung. Suy ra tip tuyn ti P v Q
phi ct nhau ti S.
V SP2 = SQ2 nn S thuc trc ng phng ca (O1) v (O2), chnh l ng thng AM,
hay S thuc mt ng thng c nh.
Bi 2.
t s(k,h, n) = 11(k + h) + 10(nk + nh). Do s(h,k) = s(k, h) nn ta lun gi s k h.
Ta thy nu n m th s(k,h, n) s(k,h,m) mod 110, do ta ch cn tm cc n 11.
Xt s(6,6,n) s(1,1,n) = 110 + 20n(n5-1). Do nu n(n5-1) chia ht cho 11 th n khng
tha mn iu kin.
T y ta loi cc gi tr n = 1, 3, 4, 5, 9, 11.
Xt s(8,2,n) s(6,4,n) = 10(n8-n6+n2-n4) = 10(n6-n2)(n2-1). Do nu n2 1 chia ht cho
11 th n khng tha mn iu kin. Vy ta loi gi tr n = 10.
Ta chng minh vi n = 2, 6, 7, 8 th s(k,h,n) s(k,h,n) khng chia ht cho 110 vi mi
b {k, h} {k,h}.
Trc ht bng cch th trc tip, ta thy rng vi n = 2, 6, 7, 8 th nk nh mod 11 vi
mi k h. (*)
Tht vy, nu s(k,h,n) s(k,h,n) chia ht cho 110 th 11(k+h-k-h) + 10(nk+nh-nk-nh)
chia ht cho 110, suy ra
k+h k h 0 mod 10 (1)
v
nk + nh nk nh 0 mod 11 (2)
T (1) suy ra k k h h (mod 10). T y, theo nh l nh Fermat, ta c nk-k nh-h
(mod 11)
Vit (2) li thnh nk(nk-k-1) nh(nh-h-1) (mod 11).
Theo l lun trn th nk-k-1 nh-h-1 mod 11. Theo (*) th nk-k-1 nh-h-1 0. Nh vy
ta c th chia hai v cho nk-k-1 c nk nh (mod 11). T (*) suy ra k = h. T (1) suy
ra k = h. Nh th {k,h} = {k, h}. Ta c iu phi chng minh.
Nhn xt. C th a ra c trng ngn gn cho cc s n tha mn iu kin l: n phi l
cn nguyn thy modulo 11. Bi ton c th tng qut ha bng cch thay 11 bng 1 s
nguyn t bt k (v d nhin 10 c thay bng p-1 v 110 c thay bng p(p-1).
y l mt bi ton kh p. tng n gin nhng khng tm thng.
71 | Trn Nam Dng 6/2010

Vietnamese IMO Team Training Camp 2010


Bi 3. Ta chng minh s hnh ch nht n nh nht cn s dng l 1006.
Trc ht ta ch ra cch ghp kht to thnh hnh ch nht 2008 x 2010 vi 1006 hnh ch
nht n (v cc hnh ch nht kp).

Khi 1
Khi 2
Khi 3
Khi 4
Trn hnh v m t cch ghp hnh ch nht 10 x 16. Hnh ch nht 2010x2008 c th
c to thnh t cu hnh trn nh sau:
+ Thm dng bng cch chn thm cc khi c dng

vo gia cc khi hp thnh t 4 ct lin tip. Mi ln thm c 2 dng.


+ Thm ct bng cch lp li cc khi 4 ct lin tip (ch tnh tun hon ca cc khi
ny: Khi 1 ~ Khi 3, Khi 2 ~ Khi 4 ).
+ Theo nh cch ghp trn th ta chia hnh ch nht 2010x2008 thnh 502 khi, mi khi
gm 4 ct. khi 1 v khi 502 ta cn dng 3 hnh ch nht n (tng ng vi cc khi
1 v 4 nh trong v d trn). Cc khi cn li ta dng 2 hnh ch nht n.
Nh vy tng cng trong cch ghp trn ta dng 500 x 2 + 2 x 3 = 1006 hnh ch nht
n.
Xoay hnh ch nht 2010x2008 li, ta c hnh ch nht 2008 x 2010.
By gi ta chng minh phi cn t nht 1006 hnh ch nht n ph hnh ch nht
2008 x 2010. Xt mt php ph hp l, gi x, y, z, t ln lt l s hnh ch nht 1 x 2, 2 x
1, 2 x 3, 3 x 2 s dng trong php ph ny. T trng cc hng l, t en cng hng chn.
in s i vo hng th i. Ta c
Nhn xt 1. 2(x+y) + 4(z+t) = 2008 x 2010. (*)

72 | Trn Nam Dng 6/2010

Vietnamese IMO Team Training Camp 2010


Nhn xt 2. Xt trn ton bng, hnh ch nht khuyt v hnh ch nht 2 x 1 c s trng
bng s en. Suy ra s hnh ch nht 1 x 2 cc hng trng = s hnh ch nht 1 x 2
cc hng en = x/2. Nh vy x chn.
Nhn xt 3. Vi mi hnh ch nht ca php ph, ta xt tng cc s trong en tng
cc s trong trng. Khi i vi hnh ch nht 3 x 2 i lng ny = 0, vi hnh ch
nht 2 x 3 i lng ny bng 2, vi hnh ch nht 2 x 1 i lng ny bng 1. Cui
cng, tng cc s trong hnh ch nht 1 x 2 l mt s chn thuc [2, 2.2008]. Do ta c
bt ng thc
Tng cc s en Tng cc s trng = 1004.2010 (x/2)(2.2008-2) + y +
2z.

1004.2010 2007x + y + 2z. (**)
i ch hng v ct, ta c
1005.2008 2009y + x + 2t (***)
Cng (**) vi (***) ri (*), ta c
1005.2008 2007x+2009y 2009(x+y)
Suy ra x + y 1005.2008/2009 > 1004. V x, y chn nn x + y 1006.
Nhn xt.
1. y l bi ton kh nht ca ngy th nht v cng l bi ton kh nht ca c k thi.
im mu cht ca li gii l tm ra cch ph ti u v chng minh tnh ti u ca n.
Vic tm ra cch ph cho hnh ch nht 4 x 2n vi 4 hnh ch nht n l khng kh
nhng rt d dn n ng nhn l vi hnh ch nht 4m x 2n (2m < n) ta c th nhn cch
ph trn thnh cch ph ti u s dng 4m hnh ch nht n. Trong thc t, cch ph
ti u s dng 1 hnh ch nht kp thay hai hnh ch nht n cho cc khi k nhau.
2. Sau khi tm ng cch ph ti u, ta cn chng minh tnh ti u ca n. Mt cch
tip cn truyn thng l t mu. Tuy nhin, mt vi php th v sai cho thy cch t mu
en trng hoc A, B, C, D n gin khng gii quyt c vn v ta phi s dng n
nhng mi lin h su sc hn gia cc v s ha cc mi lin h ny bng cch a
trng s vo.
3. C kin cho rng bi s 3 ging vi mt bi ton thi chn i tuyn Vit Nam nm
1993. Tuy nhin, nu xem xt k li gii th s ging nhau ch l hnh thc. Bi 3 nm
nay kh hn rt nhiu so vi bi 1 nm 1993.
(VTST 1993, Bi 1) Gi hnh ch nht kch thc 2 x 3 (hoc 3 x 2) b ct b mt hnh
vung 1 x 1 mt gc l hnh ch nht khuyt n. Gi hnh ch nht kch thc 2 x 3
(hoc 3 x 2) b ct b hnh vung 1 x 1 hai gc i din l hnh ch nht khuyt kp.
Ngi ta ghp mt s hnh vung 2 x 2, mt s hnh ch nht khuyt n v mt s hnh
ch nht khuyt kp vi nhau, sao cho khng c hai hnh no chm ln nhau, to
thnh mt hnh ch nht kch thc 1993 x 2000. Gi s l tng s cc hnh vung 2 x 2
73 | Trn Nam Dng 6/2010

Vietnamese IMO Team Training Camp 2010


v hnh ch nht khuyt kp cn dng trong mi cch ghp hnh ni trn. Tm gi tr ln
nht ca s.
Li gii bi ton trn kh n gin (khng phi ngu nhin bi ny c t v tr bi
s 1 ca k thi): T mu cc hng xen k en trng th s c 997 hng en v 996 hng
trng, suy ra s en nhiu hn s trng l 2000. Mi mt hnh vung 2 x 2 v mi
mt hnh ch nht khuyt kp lun c ng 2 trng, 2 en, cn mi mt hnh ch nht
khuyn n c s en tr s trng = 1 hoc -1. V vy nu gi x, y, z ln lt l s
hnh vung 2 x 2, s hnh ch nht khuyt kp, s hnh ch nht khuyt n th ta ln
lt c:
1) 4x + 4y + 5z = 1993 x 2000 (s )
2) 2000 z (s en s trng)
T y suy ra
4x + 4y 1988 x 2000
Suy ra s = x + y 994000. Php ph ti u c th xy dng d dng nh cu hnh c bn
sau:

Bi 4.
p dng bt ng thc AM-GM, ta c

suy ra

Cng bt ng thc ny vi hai bt ng thc tng t, ta suy ra

Hn na, ta li c
V vy,
74 | Trn Nam Dng 6/2010

Vietnamese IMO Team Training Camp 2010

(1)
By gi, s dng bt ng thc c bn

ta suy ra

tc
(2)
Kt hp (1) v (2), ta c ngay kt qu cn chng minh. ng thc xy ra khi v ch khi
Nhn xt.
nh gi

l kh c bn v t nhin, ng

thi cng l bc mu cht n gin ha bt ng thc.


Sau khi tm c nh gi

, ta s dng iu kin

mt cch trc tip a bt ng thc cn chng


minh v dng thun nht

n y, ta c nhiu phng hng chng minh khc nhau, trong c cch trnh by
trong li gii.
Bi 5.
Cch 1. Ta nhn xt rng vi iu kin ca bi th vi mi h thuc {1, 2, , n} th tp
hp cc i biu h nhm bt k s n t t nht h nc. Tht vy, h nhm gm hk i
din, m mi nc ch c k i biu suy ra s nc c i din trong h nhm ni trn
khng th di h nc.
Ta gi tnh cht h nhm bt k c i biu ca t nht h nc l tnh cht (*). chng
minh bi ton, ta chng minh rng ta c th chuyn bt cc i din ra khi cc nhm sao
cho
1) Mi nhm ch cn li mt i din
2) iu kin (*) vn c tha mn
R rng nu khi ta chn cc i din cn li trong phng th chnh l n ngi tha
mn yu cu bi ton, v theo iu kin (*) vi h = n th cc i din ny s l i biu
n t n nc.
Ta chng minh khng nh ni trn bng cch s dng nguyn l cc hn. Trong cc
cch chuyn bt cc i din ra khi cc nhm sao cho iu kin (*) vn c tha mn,
chn cch chuyn c s i din c chuyn ln nht. Ta chng minh rng vi cch
chuyn ny, mi nhm ch cn li 1 i din.
75 | Trn Nam Dng 6/2010

Vietnamese IMO Team Training Camp 2010


Tht vy, gi s c 1 nhm no no , chng hn nhm 1 cha t nht 2 ngi x, y.
Nu ta b i ngi x khi phng ny th theo cc chn cch chuyn trn, iu kin (*)
s khng cn c tha mn. Tc l tn ti q nhm i1, i2, , iq sao cho trong cc nhm
1 ( b ngi x), i1, i2, , iq
(1)
ch cha nhiu nht i din ca q nc.
Tng t, tn ti p nhm j1, j2, , jp sao cho trong cc nhm
1 ( b ngi y), j1, j2, , jp
(2)
ch cha nhiu nht i din ca p nc.
Trong cc ch s i v j, gi s c r ch s trng nhau: i1 = j1, , ir = jr cn cc ch s kia
khc nhau. Khi do cc nhm i1, i2, , ir cha i din ca t nht r nc nn c t nht
r nc trng nhau hai danh sch (1) v (2). Do , hai danh sch (1) v (2) gp li cha
nhiu nht i din ca q + p r nc (3).
Mt khc, hp hai danh sch cc phng ni trn li, ta c danh sch cc phng phn
bit
1, i1, i2, , ir, ir+1, , ik, jr+1, , jp
(nhm 1 thiu x v nhm 1 thiu y hp li thnh nhm 1).
p dng iu kin (*) ta thy hp ca hai danh sch ny cha i din ca t nht 1 + q +
p r nc, mu thun vi (3).
Bi ton c gii quyt hon ton.
Cch 2. li gii t nhin, chng ti trnh by thm phn dn dt. Trong li gii c th
b i phn in nghing.
thun tin trnh by, ta pht biu bi ton di dng tp hp: Cho A1, A2, , An l n
tp con k phn t ca X = {1, 2, ,n}, trong mi phn t xut hin ng k ln. Khi
tm c a1, a2, , an i mt khc nhau sao cho ai thuc Ai.
D thy hp ca p tp hp bt k cha t nht p phn t.
tng l ta s chn a1 mt cch bt k t tp A1, a2 l mt phn t khc a1 chn t A2.
Ta c chn ngu nhin theo nguyn tc nh th cho n khi khng chn c na. Tc l
ta gp trng hp gp tp hp Ap+1 trong tt c cc phn t ca n c chn
trc . Gi s ai thuc Ai, i = 1, 2, , p l cc phn t c chn.
Ta t J1 = { i {1, 2, , k}| ai Ak+1}. Nu
th ta
chn a thuc B. Gi s a thuc Ai1 th theo nh ngha, ai1 thuc Ak+1, khi ta i li,
chn a t Ai1 cn ai1 t Ak+1. Nh vy ta m rng cch chn cc phn t phn bit
n Ak+1.
Trong trng hp B1 = , ta li t J2 = {i {1, 2, , k} |
}. Nu
th ta chn a thuc B2. Gi s a thuc Ai1 th ai1 s thuc
Ai2 vi i2 no thuc J1, suy ra do ai2 thuc Ak+1. By gi ta chn a t Ai1, ai1 t Ai2
v ai2 t Ak+1. Nh vy ta m rng cch chn cc phn t phn bit n Ak+1.

76 | Trn Nam Dng 6/2010

Vietnamese IMO Team Training Camp 2010


Nu B2 = th ta li tip tc thc hin qu trnh trn .
By gi vo li gii chnh.
Ta xt th c hng G gm p+1 nh: 1, 2, , p, p+1. nh i c ni n nh j nu
ai thuc Aj. Ta t
J = { i {1, 2,, p+1}| c ng i t i n p+1}
Vi mi i {1, 2,, p}, ta c ai A  ai thuc Aj vi j no thuc J  c cnh ni
t i n j J  G cha ng i t i n p+1  i J.
T {i | i {1, 2,, p}, ai A} = J \ {p+1} = J*.
t B = A \ {ai| i J*}. Khi B {a1, a2,, ap} = v |B| = |A| - |J*| = |A| - |J| + 1
1.
Xt a B. Khi a Aj vi j J no . R rng j p+1. Nh vy c 1 ng i t j
n p+1. Gi s l j0=j, j1,, jl = p+1. Ch rng I = {j0, j1,, jl} J. By gi nu
t bj0, bj1, , bjl tng ng l a, aj0, , aj(l-1) th r rng bi Ai vi mi i thuc I. t
bp+1 = ajl, bi = ai vi mi i I. Khi b1, b2, , bk+1 l cc phn t phn bit ly t A1,
A2, , Ap+1 tng ng.
Nh vy ta m rng c cho p+1 tp hp. Bi ton c chng minh hon ton.
Cch 3. Li gii ny s dng nh l Hall sau y:
B : Cho A1, A2, , An l mt h cc tp con ca mt tp hp X tha mn iu kin
vi mi I X.
(*)
|
Khi tn ti a1, a2, , an phn bit sao ca X sao cho ai Ai vi mi i = 1, 2, , n.
Chng minh.
Ta chng minh bng quy np theo n. Vi n = 1, kt lut ca b l hin nhin.
Gi s b ng vi mi h tp con F c |F| < n. Xt h F = (A1, A2, , An). Ta xt hai
trng hp.
Trng hp 1: Tn ti mt tp ch s I {1, 2, , n}, | I | < n sao cho |
.
, Aj* = Aj \ A vi mi j J. Ta chng minh h cc
t J = {1, 2, , n} \ I,
Aj* tha mn iu kin (*). T p dng gi thit quy np cho cc h Ai, i I, Aj*, j
J ta suy ra iu phi chng minh.
Gi s ngc li, tn ti K J sao cho |
Khi , do
nn ta c
|
|
Mt khc, do
nn v tri ca ng thc trn
|
Mu thun.
77 | Trn Nam Dng 6/2010

bng

Vietnamese IMO Team Training Camp 2010

Trng hp 2: Vi mi I {1, 2, , n}, | I | < n, ta c |


. Khi chn mt
phn t bt k a1 t A1. t Ai* = Ai \{a1} vi mi i = 2, 3, , n. Khi vi mi I { 2,
, n}, ta c
|
Nh vy h (A2*, , An*) tha mn iu kin (*) v theo gi thit quy np, ta c th
chn c cc phn t phn bit a2, a3, , an sao cho ai Ai* vi mi i = 2, , n. Kt
hp vi a1 ta c cc phn t phn bit a1, a2, , an trong ai Ai.
Quay tr li bi ton, bng cch m hnh ha cc i din ca cc nc nh cc phn t,
cc nhm nh cc tp con, bng cch l lun tng t nh li gii 1, ta thy cc tp con
ny tha mn iu kin (*) v do tm c cch chn tha mn yu cu bi ton.
Nhn xt. Vi k=1 v k = n, kt lun ca bi ton l hin nhin. Vi k = 2, ta c mt
cch gii kh n gin nh sau: nhm 1, ta ly ra 1 i din bt k thuc nc i1, tip
theo, ta n nhm c ngi cn li ca nc i1, ly ngi cn li trong nhm (thuc
nc i2) lm i din cho nhm ny, li chuyn sang nhm c ngi cn li ca nc i2
Nu qu trnh ny c th ko di mi n ht n nhm th xong. Nu khng s xy ra
trng hp sau khi chn i din cho nhm k thuc nc ik th ngi cng nc vi cn
li trong nhm ny li thuc nhm 1. Nh th cc nhm 1, 2, , k to thnh mt xch, ta
loi xc ny i v lm vic vi nhng nhm cn li bng cch tng t.
Bi 6.
Ta c .
So snh h s ca

2 bn ta c
.

Tip theo ta chng minh

khng chia ht cho .

Gi s 2n c biu din tam phn l

vi ai {0, 1, 2}.
Trng hp 1: cc ai thuc tp {0, 1}, Khi a0 + a1 + + ak = 2p.
4n c biu din tam phn l
(do ai = 0 hoc 1) v theo nh l Lucas, ta
c

Trng hp 2: tn ti i nh nht m ai = 2, khi h s tng ng ca 4n l 1. Do


78 | Trn Nam Dng 6/2010

Vietnamese IMO Team Training Camp 2010

nn

Nhn xt.
V nh l Lucas khng c s dng nn th sinh khi s dng phi chng minh
li nh l ny.
Cng c th trnh by li gii trc tip khng thng qua nh l Lucas, tuy nhin
c l khng th trnh khi vic s dng h tam phn v l lun t hp di y.
Cch tip cn ca bi, yu cu chng minh S2n + 1 khng chia ht cho 3 c hai
mc ch: 1) Kim tra xem th sinh c tnh c S2n khng? V y cng l ch
cho im.
2) a ra bn cht t hp ca

t gi n cch khai trin a thc

theo m-un.
* hon chnh li gii, chng ti trnh by cch chng minh nh l Lucas.
nh l Lucas. Cho m v n l cc s nguyn khng m, p l s nguyn t v
m = mkpk + mk-1pk-1 + + m1p + m0
n = nkpk + nk-1pk-1 + + n1p + n0
l cc biu din p phn ca m v n tng ng. Khi
.
Chng minh. Ta lm vic trn cc a thc vi h s c xt theo modulo p. Do
vi mi k = 1, 2, , p-1 nn ta c
(1+ x)p 1 + xp (mod p)
T bng quy np ta suy ra
(mod p)
Nh vy ta c
(mod p)
H s ca xn v tri l
v phi l

. Do biu din p phn ca n l duy nht nn h s ca xn

T y ta c iu phi chng minh.

Li gii trn y c s dng p n chnh thc, li gii v tng ca cc bn LTL,


Phm Minh Khoa, V Quc B Cn, Traum v mt s tho lun khc trn din n
www.mathscope.org.
79 | Trn Nam Dng 6/2010

Vietnamese IMO Team Training Camp 2010

Bt ng thc: Mt s v d v bi tp chn lc
1. (USA MO 2004) Cho a, b, c l cc s thc dng. Chng minh rng
(a5 a2 + 3)(b5 b2 + 3)(c5 c2 + 3) (a+b+c)3
2. (IMO 2005) Chng minh rng nu a, b, c l cc s dng c tch ln hn hay bng 1
th
a5 a2
b5 b 2
c5 c 2
0
+
+
a 5 + b 2 + c 2 b5 + c 2 + a 2 c5 + a 2 + b 2

3. (Kvant) Cho a, b, c l cc s thc dng tho mn iu kin a + b + c = 1. Chng minh


rng ta lun c
1 1 1
+ + + 48(ab + bc + ca ) 25
a b c

4. (Mathlinks) Cho a, b, c, x, y, z l cc s thc tha mn iu kin


(a+b+c)(x+y+z) = 3, (a2+b2+c2)(x2+y2+z2) = 4
Chng minh rng ax + by + cz 0.
5. (Vit Nam 2002) Cho x, y, z l cc s thc tho mn iu kin x2 + y2 + z2 = 9. Chng
minh rng 2(x+y+z) xyz 10.
6. Cho x, y, z l cc s thc tho mn iu kin x + y + z = 0 v x2 + y2 + z2 = 6. Tm gi
tr ln nht ca biu thc F = x2y + y2z + z2x.
7. (Vasile Cirtoaje) Cho cc s thc khng m tha mn iu kin a + b + c = 4. Chng
minh rng
a2b + b2c + c2a + abc 4.
8. (IMO 1999) Cho n 2 l mt s nguyn dng c nh, tm hng s C nh nht sao
cho vi mi s thc khng m x1, x2, , xn
n
xi x j ( x + x ) C x i

1i < j n
i =1
2
i

2
j

9. Cho a, b, c l cc s thc dng cho trc v x, y, z l cc s thc dng thay i


tho mn iu kin xyz = ax + by + cz. Chng minh rng gi tr nh nht ca x + y + z
bng b + c + 2bc / d + c + a + 2ca / d + a + b + 2ab / d , trong d l s thc dng xc
nh bi phng trnh
a
b
c
+
+
= 1.
a+d b+d c+d

80 | Trn Nam Dng 6/2010

Vietnamese IMO Team Training Camp 2010


10. (Romanian TST 2007) Cho n 2 v x1, , xn ; y1, , yn l 2n s thc tho mn iu
kin
n

ai2 = 1,
i =1

bi2 = 1, ai bi = 0,
i =1

i =1
2

Chng minh rng ai + bi n.


i =1

i =1

11. (IMO Shortlist 2007) Cho a1, a2, , a100 l cc s thc khng m tho mn iu kin
a12 + a22 + + a1002 = 1. Chng minh rng
a12a2 + a22a3 + + a1002a1 < 12/25.
12. IMO Short List 2003) Cho n 2 l s nguyn dng v x1, x2, , xn, y1, y2, , yn l
2n s thc dng. Gi s z2, z3, , z2n l cc s thc dng sao cho z2i+j xiyj vi mi i,
j thuc {1, 2,,n}. t M = max{z2, z3,, z2n}. Chng minh rng
M + z 2 + z 3 + ... + z 2 n

2n

x1 + ... + x n y1 + ... + y n

n
n

13. Cho a1, a2, , an l cc s thc sao cho a12 + a22 + + an2 = 1. Tm gi tr ln nht
ca biu thc a1a2 + a2a3 + + an-1an.
14. Cho x1, x2, , xn l cc s dng. Gi A l s nh nht trong cc s
x1 , x 2 +

1
1
1 1
, x3 + ,..., x n +
, , cn B l s ln nht trong cc s ny. Chng minh
x1
x2
xn 1 x n

rng gi tr ln nht ca A bng gi tr nh nht ca B v hy tm gi tr ny.


15. Tng n s thc dng x1, x2, x3, ..., xn bng 1. Gi S l s ln nht trong cc s
x1/(1 + x1), x2/(1 + x1 + x2), ..., xn/(1 + x1 + x2 + ... + xn). Tm gi tr nh nht ca S. Vi
nhng gi tr no ca x1, x2, x3, ..., xn th gi tr ny t c?

81 | Trn Nam Dng 6/2010

You might also like